GENERAL STUDIES FOR ALL UPSC/PSC EXAMS Headline Animator

Flipkart.com

Wednesday, January 25, 2012

UPSC CIVIL SERVICES MAINS EXAM SOLVED PAPER 2011

General Studies Paper – I


1. Answer any three of the following in about 250 words each:         20x3=60

(a) ‘Essentially all that is contained in part IV- A of the Constitution is just a codification of tasks integral to Indian way of life.’ Critically examine the statement.

Answer:

Article 51A of the part IV-A of the Indian Constitution lists the fundamental duties of the citizens which were added to the Indian Constitution by the 42nd Amendment Act. Fundamental duties are restrictions on the citizens, but they are not enforceable in a court of law. They act more like a lighthouse to guide citizens’ conduct and bring it in conformity with the Indian way of life. They include abiding the constitution and respecting its ideals and institutions such as the National Flag and the National Anthem. Fundamental Duties also include cherishing and following the noble ideals that inspired our freedom struggle, upholding the sovereignty and integrity of the nation, rendering national services, promoting harmony and brotherhood, renouncing practices derogatory to the dignity of women, safeguarding public property, developing scientific temper etc.

Incidents like destruction of public property by violent mobs and protestors, delivering of hate speeches to cause disharmony and rift among communities, mounting corruption, declining child sex ratio, reports of practices like sati which still is continuing in some parts of the country point towards the fact that the republic has not succeeded completely in instilling the values contained in part IV-A, in the hearts and minds of the Indian citizens.

These values should be taught from the early childhood through a free, fair, secular, and non-discriminatory education system. The society also needs role models from all walks of life such as politics, business, administration, judiciary, academia etc.  so that national identity becomes paramount and the values are most cherished.


(b) ‘The exercise of executive clemency is not a privilege but is based on several principles, and discretion has to be exercised in public consideration.’ Analyse this statement in the context of judicial powers of the President of India.

Answer:

Article 72 of the Indian Constitution empowers the president to pardon, remit, commute, respite and reprieves a person of any offence. Supreme Court has held that pardoning power of the President is subject to judicial review and it should not be handled dishonestly in the public interest.

The question of executive clemency has come into focus due to the recent decision of the President’s rejecting the mercy plea of those, convicted in the Rajiv Gandhi assassination case and Tamil Nadu assembly’s passage of a resolution over it. The Afzal Guru case has also not yet been resolved which also is giving political colour to the whole issue.

Supreme Court in its 1989 judgement laid down several principles or ‘do’s’ and ‘don’ts’ with respect to the executive clemency. The apex court observed that the delay in making a decision on the death penalty leads to adverse psychological impact on the convicted and it amounts to the court’s inhuman and brutal treatment. Thus inordinate delay can form the basis for clemency. It also observed that the nature of crime needs to be taken into consideration before granting executive clemency. The conduct of the convicted cannot form the basis for granting clemency and the time calculated should be from the date the final verdict was given on the case if it needs to form the ground for clemency.

Constitution should be amended to provide the time limits within which mercy petition are to be decided. Importantly, the political parties should restrain from politicising the power of the President which is supposed to be used in the public consideration.


(c) Discuss the extent, causes, and implications of ‘nutrition transition’ said to be underway in India.

Answer:
Nutrition Transition can be referred to as the increased consumption of unhealthy foods compounded with increased prevalence of overweight in middle-to-low-income countries. It has serious implications in terms of public health outcomes, risk factors, economic growth and international nutrition policy.

Extent: As developing societies like India industrialise and urbanise, and as standards of living continue to rise, weight gain and obesity are beginning to pose a growing threat to the health of the citizens. Repeated episodes of malnutrition, followed by nutritional rehabilitation, are known to alter body composition and increase the risk of obesity. Food balance data from the Food and Agriculture Organization (FAO) show that the change in energy intake in Asian countries has been small, but there have been large changes in consumption of animal products, sugars and fats in countries like India. There is a progressive increase in the intake of protein, and probably fats. The increase in the intake of protein and fats is due to the phenomenal increase in the consumption of milk and milk products and an increase in the intake of animal products. On the other hand consumption of pulses and legumes has fallen drastically in India.

Causes: In India, the demographic and epidemiological transition, the forces of internal migration and urbanisation, the changes in food consumption patterns and low physical activity patterns to an epidemic of obesity and other NCDs (Non-communicable Diseases). There is also a  decrease in the energy expenditure in occupational activities, increased urbanisation, universal use of motor cars, mechanisation of most manual jobs outside the occupational sphere and increasing leisure time have aggravated this trend in India.

Implications: There is a large increase in the non-communicable diseases (NCDs) in the developing countries specially the countries under transition like India. Approximately 40% of the deaths in the developing countries take place due to NCDs.

(d) Bring out the salient features of the PCPNDT Act, 1994, and the implication of its amendment in 2003.

Answer:
Pre Conception and Pre-Natal Diagnostic Technique Act, 1994 was enacted to arrest the declining sex ration. It is a subject of discussion now because; an all-time low child sex ratio of 914 was reported in the 2011 provisional census data.

The main purpose of enacting the act is to ban the use of sex selection techniques before or after conception and prevent the misuse of prenatal diagnostic technique for sex selective absorption.

Offences under this act include conducting or helping in the conduct of prenatal diagnostic technique in the unregistered units, sex selection on a man or woman, conducting PND test for any purpose other than the one mentioned in the act, sale, distribution, supply, renting etc. of any ultra sound machine or any other equipment capable of detecting sex of the foetus.

The act was amended in 2003 to improve the regulation of the technology used in sex selection.

Implications of PCPNDT Act, 1994 amendment:

1. Amendment of the act mainly covered bringing the technique of pre conception sex selection within the ambit of the act

2. Bringing ultrasound within its ambit

3. Empowering the central supervisory board, constitution of state level supervisory board

4. Provision for more stringent punishments

5. Empowering appropriate authorities with the power of civil court for search, seizure and sealing the machines and equipments of the violators

6. Regulating the sale of the ultrasound machines only to registered bodies

2. Answer one of the following in about 250 words:        20x1=20

(a) Trace the salient sequences of events in popular revolt that took place in February 1946 in the then Royal Indian Navy and bring out its significance in the freedom struggle. Do you agree with the views that the sailors who took part in this revolt were some of the unsung heroes of the freedom struggle?

Answer:

Royal Indian Navy revolt of February 1946 took place in the background of Quit India Movement and Second World War. This was a very turbulent phase in India’s freedom struggle. The popular revolt shook the very foundation of British Raj and made it abundantly clear that their time in India was numbered.
In November 1945 some students from Forward Block, Students Federation of India and Islamia College participated in a protest march over the INA trials. They tied together League, Congress and red flag, as a symbol of anti imperialist unity.

In February 1946, Muslim League students took a protest march in which some Congress students also participated against the seven year sentence to INA prisoner Rashid Ali.

In February 1946, naval ratings of HMIS Talwar went on strike to protest against racial discrimination, unpalatable food, INA trials, and abuse by superior officers. This was followed by city people joining in through mass strikes, hartals, meetings, attacks on police stations, railway station etc. Other parts of the country also expressed support in the form of strikes by Royal Indian Forces in Calcutta, Puna and Bombay.

The upsurge showed that the fearless action by the masses, revolt in armed forces had psychological affect on masses and it also prompted British to extend some concessions but above all it marked the end of British rule in India.

Sailors who took part in the struggle were the unsung heroes as they did not get the level of publicity as that of the INA trials and in the pages of history; they remain anonymous and unknown.

(b) Evaluate the influence of three important women’s organisations of the early twentieth century in India on country’s society and politics. To what extent do you think were the social objectives of these organisations constrained by their political objectives?

Answer:

Bharat Stree Mahamandal, All India Women’s Conference and Women’s India Association were some of the important women’s associations of the early twentieth century. Bharat Stree Mahamandal was the first women's organisation in India founded by Sarala Devi Chaudhurani in Allahabad in 1910. One of the primary goals of the organisation was to promote female education which was not well developed at that time. The organisation opened several offices in Lahore, Allahabad, Delhi, Karachi, Amritsar etc. to improve the condition of women all over India.

All India Women’s Conference was founded in 1927 by Margret cousins having Sarojni Naidu, Lady Dorab Tata as its founding members. It worked towards women’s education, abolition of purdah system, legislative reform, abolition of child marriage, harijan welfare, family planning, and rural reconstruction. These women’s organisations worked for a society based on principles of social justice, integrity, equal rights and opportunities.  They wanted security for every human being; the essentials of life not determined by accidental births but by planned social distribution.

Their efforts led to several legislative reforms in Sharda Act (1929), Hindu Women’s Right to Property Act (1937), Factory Act (1947), Hindu Marriage and Divorce Act etc. AIWC efforts also led to setting up of The All India Women’s Education Fund
Association, and Lady Irwin College of Home Science.

Social and educational reforms effort by the women’s associations helped in preparing the Indian women to participate in the freedom struggle. With Mahatma Gandhi women availed an opportunity to get into the scene of freedom struggle.

3. Answer any one of the following in about 250 words:    20x1=20

(a) Critically examine the design of National Rural Livelihood Mission (NRLM) scheme. Do you think it has a better chance of
success than the Swarna Jayanti Swarojgar Yojna(SGSY) in achieving its objectives?

Answer:

NRLM is the rechristened version of Swarna Jayanti Swarojgar Yojna (SGSY). It is a Ministry of Rural Development programme. It aims to reduce poverty by enabling the poor household to access gainful self employment and skilled wage employment opportunity resulting in a sustainable livelihood.

NRLM is based on three pillars

1. Enhancing and expanding the existing livelihood options of the poor

2. Building skills for the job market

3. Nurturing self employed and entrepreneurs

NRLM plans to give special focus on the poorest households who are currently dependent on the MGNREGA. The design of
NRLM is more likely to succeed because its implementation is in a mission mode which enables it shift from the present allocation based approach to demand based approach. This enables the states to formulate their own livelihood based on poverty reduction action plans. It also focuses on targets, outcomes, and time bound strategy. The monitoring would be done against the targets of the poverty outcomes. NRLM will have continuous capacity building, imparting of requisite skills and creating linkages with livelihood opportunities for the poor, including those emerging from the organised sector.

NRLM funds will be directly released to the state level agencies and DRDA based on the detailed district wise annual action plan.  It will involve the Self Help Group in the implementation which increases the likelihood of its success. National Skill Development Council will also coordinate in the skill development part in the implementation of NRLM. In order to ensure institutional arrangement for skill development for self employment and wage employment, dedicated training institute for rural BPL youth i.e Rural Self Employment Training Institutes (RSETIs) are being set up with the aim of having at least one such institution in each district in India. These RSETIs will be set up with the partnership of banks. This will help in achieving the objectives of NRLM.

(b) Highlight the structure, objectives and role of the Advertising Standard Council of India. In what way has the August 2006
government notification made it more effective?

Answer:

The Advertising Standards Council of India (ASCI), established in 1985, is committed to the cause of Self-Regulation in Advertising, ensuring the protection of the interests of consumers. The ASCI was formed with the support of all four sectors connected with Advertising, viz. Advertisers, Ad Agencies, Media (including Broadcasters and the Press) and others like PR Agencies, Market Research Companies etc. Its main objective is to promote responsible advertising; thus enhancing the public's confidence in Advertising. ASCI thus aims to achieve its own overarching goal i.e., to maintain and enhance the public's confidence in advertising.

The Board of Governors (16 members) ensures equitable representation of Advertisers, Agencies, Media and other Advertising Services, the individual member firms being leaders in their respective industries or services. The Consumer Complaints Council (CCC) (21 members) has 12 Non-Advertising professionals representing civil society, who are eminent and recognised opinion leaders in their respective disciplines such as Medical, Legal, Industrial Design, Engineering, Chemical Technology, Human Resources and Consumer Interest Groups; 9 are advertising practitioners from our member firms.

ASCI is represented in all committees working on advertising content in every Ministry of the Government of India. ASCI’s Code for Self-Regulation in Advertising is now part of ad code under Cable TV Act’s Rules. Violation of ASCI’s Code is now treated as a violation of the government’s rules. ASCI’s membership of The European Advertising Standards Alliance (EASA) ensures that it gets valuable advice, learning and even influence at the international level.
The Consumer Complaints Council is ASCI's heart and soul. It is the dedicated work put in by this group of highly respected people that has given tremendous impetus to the work of ASCI and the movement of self-regulation in the advertising.

In August 2006, the Ministry of Information and Broadcasting issued a notification deeming it necessary for all TV commercials in India to abide by the ASCI code. This effort of the Ministry of Information and Broadcasting has made the advertising self-regulatory movement in India stronger and more effective.


4. Comment on any five of the following in about 150 words each.    12x5=60

(a) Salient recommendations of the RBI-appointed Damodaran committee on customer service in Banks.

Answer:

The Damodaran Committee on bank customer services has recommended active involvement of the boards of banks to guarantee customer satisfaction. The committee held that customer service and grievance redress should be included as a mandatory parameter in the performance appraisal report of all employees.

The committee has suggested that an agenda on the level of implementation of the Bank's Code of Commitments to Customers and an overview on the grievance redress mechanism in the bank should be placed before the bank every quarter before the Customer Service Committee.

The committee suggets that every board should ensure they have comprehensive policies for customer acceptance, care and severance. The banks should show sensitivity for small customers by ensuring that the pricing (bank charges) does not act as a deterrent for the small person to do banking transactions.

Emphasising on 'customer centricity', the committee recommended that bank boards should evolve human resources policies which should recruit for attitude and train for skills.

(b) Deendayal Disabled Rehabilitation Scheme (DDRS).

Answer:

It is a scheme by the Central Ministry of Social Justice & Empowerment. Its funding pattern is up to 90 percent; it is funded by the Central Ministry. The Grant in Aid is released on the basis of recommendations received from the State Government, UTs through State Multi-Disciplinary grant-in-aid committee concerned or any other agency designated by the Ministry. Beneficiary could be an individual, a family, a community, women and Children.

The voluntary organisations are assisted in running rehabilitation centres for leprosy cured persons and also for manpower
development in the field of mental retardation and cerebral palsy. They are also assisted in establishment and development of special schools for major disability areas, viz. Orthopaedic, speech, hearing, visual and mental disability. The NGOs are extended assistance for setting up projects of vocational training to facilitate the disabled persons to be as independent as possible by acquiring basic skills. The Ministry, under the Scheme supports both recurring and non-recurring expenditure of projects by NGOs up to 90 percent.

(c) Evolution of ‘Green Benches’ in our higher judiciary.

Answer:

The Supreme Court of India interpreted Article 21 which guarantees the fundamental right to life and personal liberty, to include the right to a wholesome environment and held that a litigant may assert his or her right to a healthy environment against the State by a writ petition to the Supreme Court or a High Court. Public interest litigation has been used by the higher judiciary to ensure environment protection and safeguard public interest.

Till 1980, not much contribution was made by the courts in preserving the environment. One of the earliest cases which came to the Supreme Court of India was Municipal Council, Ratlam, vs Vardhichand AIR 1980 SC 1622. Thereafter, series of cases were filed before the Supreme Court and there was a dynamic change in the whole approach of the courts in matters concerning environment.
India has now become the third country in the world to start a National Green Tribunal (NGT) which is a judicial body exclusively meant to judge environmental cases. The National Green Tribunal has been established under the National Green Tribunal Act 2010 for effective and expeditious disposal of cases relating to environmental protection and conservation of forests and other natural resources. The Tribunal is mandated to make and endeavour for disposal of applications or appeals finally within 6 months of filing of the same. The predecessor to the NGT, the erstwhile National Environment Appellate Authority has been superseded by the NGT.

(d) Distinction between ‘Department Related Parliament Standing committees’ and ‘Parliamentary Forum’.

Answer:

Departmental Standing Committees were created in 1993 to exercise control over the executive; particularly financial control. There are now 2 such committees having 31 members each; 21 from the Lok Sabha and 10 from the Rajya Sabha. Members from the Lok Sabha are nominated by the Speaker of the Lok Sabha, while members from the Rajya Sabha are nominated by the Rajya Sabha Chairman. A minister cannot be nominated as a member of the committee. These committees consider the demand for grants of the concerned ministry. They submit the reports based on which the discussion on budget takes place.

Parliamentary Forums on the other hand are ad hoc in nature and are constituted for specific issues to make the Members of the Parliament aware of the seriousness of the particular situation and to enable them to adopt a result-oriented approach towards these issues. The Parliamentary fora do not interfere in or encroach upon the jurisdiction of the concerned Departmentally Related Standing Committees or the Ministry/Department concerned. Members of these fora are nominated by the Speaker of the Lok Sabha and the Chairman of the Rajya Sabha, as the case may be.

(e) Benefits and potential drawbacks of ‘cash transfers’ to Below Poverty Line Households.

Answer:

Cash transfer has come into discussion due to the leakages in several government schemes like the Public Distribution System. It has been successful in Latin American countries. Cash transfer has some benefits in terms of better targeting of public subsidy, reducing diversion, preventing corruption, and eliminating wastages in transportation and storage of goods like food grains. It also gives flexibility to the citizens in terms of buying the public goods and services. Cash transfer is also suitable for migrating population that moves in search of work.

Cash transfer on the other hand can only succeed if an appropriate IT infrastructure exits, through which cash can be transferred directly to the account of the beneficiary which becomes a challenge in the poor financial inclusion. Cash can be more prone to diversion if proper safeguards are not taken. Moreover cash transfer can work if the public delivery system is in place for e.g. how giving cash for health services will make any difference if there are no hospitals, medicines and doctors in the villages. Factoring inflation into cash transfer is always been a challenge.

(f) New initiatives during the 11th Five Year Plan in the National Programme for Control of Blindness (NPCB).

Answer:

The Cabinet Committee on Economic Affairs (CCEA) endorsed and approved a budget of INR 12,500 million for the XI five-year (2007-12) plan period. The enhanced funding and financial allocation to the tune of nearly two-thirds of the previous plan period is indicative of the high political commitment accorded to blindness control activities in the India. With the federal nature of the Indian Constitution, the States are largely independent in matters relating to health delivery. The Central Government’s responsibility consists mainly of policy making, planning, funding, guiding, assisting, evaluating and coordinating the work of state health ministries so that health services cover every part of the country and no state lags behind for want of these services. The NPCB is striving to enhance the capacity of health institutions, health personnel and the community at all levels to address issues under the programme. In the approved XI five-year plan period, schemes with existing/enhanced financial allocation are being implemented along with new initiatives to reduce blindness.

5. Examine any three of the following in about 150 words each.        12x3=36

(a) The impact of climate change on water resources in India.

Answer:

The impact of climate change on water resources in India is evident through erratic monsoon, more frequent floods and droughts, stronger cyclones and rivers changing their course frequently. The severity is also due to the prevailing more than 7000km of coastline.

The changed rainfall pattern has adversely affected ground water recharge, wetlands both coastal and terrestrial.  Climate change has resulted in melting of Himalayan glaciers. It has the potential of making the Himalayan Rivers swell first and then turning them into seasonal rivers, threatening the source of freshwater. It can also lead to salt water intrusion and threatening aquaculture and coastal agriculture. 

The impact has been severe also because, India is still an agricultural country with its large population being dependant on weather related livelihood through agriculture, forestry, pisciculture etc.

(b) Measures taken by the Indian government to combat piracy in the Indian Ocean.

Answer:
Indian government has used INS Talwar to foil several bids by pirates near the Gulf of Aden. Indian government is coordinating the anti piracy measures at the international level with China, European countries and with the Gulf countries.
Indian government has placed surveillance radars in countries like Maldives and Srilanka to secure the sea lanes in the Indian Ocean. It has signed agreements with Madagascar, Seychelles and Mauritius for anti piracy measures. Government has also taken measures in strengthening the coastal security by equipping and reforming the coast guards. However, incidents like unmanned vessel drifting in the ocean and reaching the coast of Mumbai have highlighted the holes in our coastal security which needs to be plugged.

Supreme Court has recently asked the Central Government to formulate separate piracy law for trying the pirates and to take care of the compensation to the victim’s family. Currently piracy is being dealt under the provisions of Indian Penal Code.
 
(c) The significance of counter-urbanisation in the improvement of metropolitan cities in India.

Answer:

Counter-urbanisation is a demographic and social process where people move from large urban areas or into rural areas, thereby leapfrogging the rural-urban fringe. It might mean daily commuting, but could also require lifestyle changes and the increased use of ICT (home working or teleworking). It is the process of migration of people from major urban areas to smaller urban settlements and rural areas.  Counter-urbanisation affects the layout of rural settlements. Modern housing estates locate of the edge of small settlements. Industrial units are sited on main roads leading into the settlement.

Counter urbanisation will reduce pressure on the metropolitan cities and its basic amenities like drinking water supply, sewage facilities, continuous supply of electricity, education facilities, etc as people move out due to following

(i) Increase in car ownership enabling their movement, growth in information technology (E-mail, faxes and video conferencing) meaning more people can work from home

(ii) Urban areas are becoming increasing unpleasant place to live. This is the result of pollution, crime and traffic congestion.

(iii) More people tend to move when they retire.

(iv) New business parks being developed on the edge of cities (on Greenfield sites) meaning people no longer have to travel to the city centre. People now prefer to live on the outskirts of the city to be near where they work.

(d) Problems specific to the denotified and nomadic tribes in India.

Answer:
Post Independence the Criminal Tribe Act 1871 was replaced; and criminal tribe nomenclature was replaced with denotified tribe (DNTs) which is still considered derogatory. They have been discriminated; British government included some of the tribes like Gonds, Ho and Santhals under this categorisation as they had rebelled against the British Raj.

A major challenge in the intervention comes due dispersed nature of the tribes and having a nomadic culture surviving on shifting cultivation. This results in health and educational services a challenging task to deliver. It also makes the implementation of nutritional initiative like addressing iodine deficiency a challenging task. Preservation of their culture, script, practices also get hampered due to the same reason. Two different opinions arise with regard to the reservation for the DNTs to raise their social and economic conditions. One view supports providing reservation within existing group of SCs, STs and OBCs while the other view supports creating a new group for the reservation of DNTs.

6. In the context of the freedom struggle, write short notes (not exceeding 50 words each) on the following:    5x3=15

(a) 'Benoy-Badal-Dinesh' martyrdom.

Answer:

On 8 December 1930, Benoy along with Dinesh chandra Gupta and Badal Gupta, dressed in European costume, entered the Writers' Building and shot dead Simpson, the Inspector General of Prisons, who was infamous for the brutal oppression on the prisoners. This inspired further revolutionary activities in Bengal. After Independence Dalhousie square was named B.B.D Bagh-after Benoy-Badal-Dinesh. 

(b) Bharat Naujavan sabha

Answer:

Bharat Naujavan Sabha was an association of Indian youths which was established at a convention held in April 1928 at the Jallianwala Bagh in Amritsar. The aims of the Sabha were to create a youth wing of peasants and workers with a view to usher in revolution in the country and overthrow the British rule. Bhagat Singh was its secretary and principle organiser.

(c) 'Babbar Akali' movement

Answer:

The Babbar Akali movement, which emerged in the wake of the Akali Movement, and was an underground terrorist movement established in the Jalandhar Doab in 1921. Members fought pitched battles with police and committed acts of violence.

7. Comment on the following in not more than 50 words each:             5x6=30

(a) Phase-IV of the tiger monitoring programme in India.

Answer:

Phase-IV of the tiger monitoring programme by by the Wildlife Institute of India (WII) and the National Tiger Conservation Authority (NTCA) means initiating intensive, annual monitoring of tigers at the tiger reserve level, across 41 protected areas in India. This programme is to estimate numbers of both tigers and their prey. This programme is planned to be commenced from November 2011.

(b) Why the Central Statistical Office has notified a new series of Consumer Price Index from this year?

Answer:

The Central Statistical Office has notified a new series of CPI with its base year of 2010 for rural, urban areas and for the nation as a whole. It will give a comprehensive picture of inflation at the national level for retail prices. Government also proposes to use it for giving dearness allowance for government employee in the seventh pay commission.

(c) Composition and functions of the National Executive Committee of the National Disaster Management Authority.

Answer:

The National Executive Committee (NEC) of the National Disaster Management Authority comprises the Union Home Secretary as the Chairperson, and the Secretaries to the GOI in the Ministries/Departments of Agriculture, Atomic Energy, Defence, Drinking Water Supply, Environment and Forests, Finance (Expenditure), Health, Power, Rural Development, Science and Technology, Space, Telecommunications, Urban Development, Water Resources and the Chief of the Integrated Defence Staff of the Chiefs of Staff Committee as members.

(d) The Bihar Special Courts Act, 2009 and why it has been in news recently?

Answer:

Bihar Special Courts Act, 2009 provides that the property of corrupt persons can be attached even when the probe is under way, if the authorised officer concludes that the acquisition of the property was the result of the offence committed by the accused. Bihar implemented it and some officials came under this Act. Its constitutional validity was questioned in Supreme Court which refused to stay this act.

(e) The Telecommunications Ministry’s proposed Spectrum Management Commission.

Answer:
Spectrum Management Commission is a new entity to manage and regulate spectrum allocation. The Commission will subsume Wireless Planning Coordination wing of the Department of Telecom and will get wider powers including dispute settlement, pricing and regulations related to spectrum. While the DoT will continue to be the licensor, all issues pertaining to spectrum allocation will be brought under Spectrum Management Commission.

(f) The Community-Led Total Sanitation (CLTS) approach to sanitation.

Answer:
CLTS is an innovative methodology for mobilising communities to completely eliminate open defecation (OD). Communities are facilitated to conduct their own appraisal and analysis of open defecation (OD) and take their own action to become ODF (open defecation free). CLTS focuses on the behavioural change needed to ensure real and sustainable improvements. It invests in community mobilisation instead of hardware, and shifting the focus from toilet construction for individual households to the creation of ’open defecation-free’ villages.

8. Attempt the following in not more than 50 words each.        5x4=20

(a) Distinguish either between the ‘Moatsu’ and ‘Yemshe’ festivals of Nagaland or the ‘Losar’ and ‘Khan’ festivals of Arunachal Pradesh.

Answer:

Moatsu Festival is celebrated in Nagaland by the Ao tribe. It is observed every year in the first week of May. During this Nagaland festival, a number of rituals are performed. After sowing the seeds, the Aos observe Moatsu Mong.

The Pochury Tribe celebrates their greatest festival, Yemshe in the month of October every year. During the Yemshe festival, the arrival of the new harvest is celebrated with full fun & fair. The Losar Festival also called as the New Year Festival and it is the most important festival celebrated in Tawang, Arunachal Pradesh. Losar is celebrated by the Monpas that forms the major portion of population in Tawang, Arunachal Pradesh.
In the Khan festival of Arunachal Pradesh, the social and cultural beliefs of the local tribes can be witnessed. Regardless the different casts and creed, the local tribes unite in the Khan celebration.

(b) Write a sentence each of any five of the following traditional theater forms:

(i) Bhand Pather: It is the traditional theatre form of Kashmir, which is a unique combination of dance, music and acting. Satire, wit and parody are preferred for inducing laughter.

(ii) Swang:
It is a popular folk dance-drama or folk theatre form in Rajasthan, Haryana, Uttar Pradesh and Malwa region of Madhya Pradesh. Swang incorporates suitable theatrics and mimicry (or nakal) accompanied by song and dialogue. Swang theatre is
traditionally restricted to men, who also play the female roles.

(iii) Maach:
It is a traditional Hindi theatre form of Madhya Pradesh. It shares the secular nature and characteristics of other north Indian genres like Nautanki and Khyal. Based on religious, historical, romantic, or social themes, it was invented and developed by prominent artists like Guru Gopalji, Guru Balmukund, Kaluram Ustad, and Radhakrishan Ustad.

(iv) Bhaona:
It is a unique festival of Vaishnava theatrical performance in Assam. The performance is marked by a continuous shifting between the classical and the folk, the mundane and the spiritual, providing thousands of spectators a rare aesthetic experience.

(v) Mudiyettu:
It is ritualistic dance drama performed after the harvest of summer crops in Kerala. In 2010, Mudiyettu was included in the UNESCO Intangible Cultural Heritage Lists.

(vi) Dashavatar:
Dashavatar is the most developed theatre form of the Konkan and Goa regions. The performers personify the ten incarnations of Lord Vishnu-the god of preservation and creativity.

(c) What are the major different styles of unglazed pottery making in India?

Answer:

There are three different styles in unglazed pottery making in India. They are - the paper-thin, scrafito and highly polished. Black pottery is another famous form of unglazed pottery in Indian villages and it resembles the Harappan pottery style. In the paper thin pottery, the biscuit coloured pottery is decorated with incised patterns.

(d) List the classical dance forms of India as per the Sangeet Natak Akademy.

Answer:
Sangeet Natak Akademi currently confers classical status on nine Indian dance styles which are Bharatnatyam, Kathak, Gaudiya Nritya, Kathakali, Mohiniyattam, Oddisi, kuchipudi, Sattriya, and Manipuri.

9. Comment on following in not more than 50 words each:        5x5=25

(a) Nisarga-Runa technology of BARC.

Answer:

The Nisarga-Runa technology developed at BARC converts biodegradable solid waste into useful manure and methane. It can be deployed for the dual objectives of waste management as well as for livelihood creation among the urban underprivileged. The 'Nisargruna' technology adopts biphasic reactor system wherein the first reactor is operated under aerobic and thermophilic conditions. As a result of the first feature, the universal problem of odour from waste processing biogas plants is eliminated and the second feature leads to a faster process. Unlike conventional single phase digesters, which take 30-40 days, a 'Nisargruna' plant can digest organic solid waste between 18-22 days.

(b) The first aid that you can safely administer to a person standing next to you at the bus stop who suddenly faints.

Answer:

I would first take the person away from the crowd to an open and safe area. Then I would sprinkle some water on his/her face to bring him/her back to consciousness. If the person does not gain consciousness then mouth to mouth respiration and administering CPR can be considered as the next step. His/her family members or friends should be immediately informed about his/her ill health by a phone call from the contact details from the mobile phone or the purse of the person and can gather information on the person’s medical condition and then accordingly he/she can be taken for medical supervision.

(c) The Kaveri K-10 aero-engine.

Answer:

Kaveri K-10 engine is being developed to be used in the Light Combat Aircraft, Tejas. It is being developed in collaboration of French firm, Snecma.  It will have less weight and more reheat thrust to meet the requirements of the Indian Army.

(d) Molecular Breast Imaging (MBI) technology

Answer:
Molecular breast imaging (MBI) is three times more effective than mammograms, and far less costly than other nuclear-medicine imaging. Molecular Breast Imaging (MBI) uses a dedicated dual-head gamma camera and 99mTc-sestamibi in women having dense breast patterns and additional risk factors for breast cancer.

(e) E-governance initiatives by the Union Public Service Commission (UPSC)

Answer:

UPSC has taken the initiative for getting the forms of several examinations like civil services filled online. It also displays the status of the application form for every candidate on its website. UPSC publishes the syllabus and tentative schedules of various UPSC conducted examinations on its website helping students to plan in advance. UPSC also publishes the various court orders and notifications on the web site and information regarding the RTI petitions on its website.

10. Who are the following and why have they been in the news recently? (each answer should not exceed 20 words): 2x7=14

(a) Lieutenant Navdeep Singh

Answer:

Lieutenant Navdeep Singh laid down his life fighting terrorists during an anti-infiltration operation along LOC in Gurez Sector of north Kashmir.

(b) Rahim Fahimuddin Dagar

Answer:

He was an exponent of Dagar vani Dhrupad, which is one of the most pristine and richest forms of Indian classical music. He represented the 19th generation of Dagar Tradition.

(c) Lobsang Sangay
Answer:

He is the Tibetan Prime Minister in exile. He is a Harvard graduate and a political successor of Dalai Lama.

(d) P.R. Sreejesh

Answer:

P.R. Sreejesh is Hockey Goalkeeper in the Indian Hockey team. He is from Kerela.

(e) Nileema Mishra

Answer:

She is one of the Magsaysay Award winners for 2011.. She is a lender to the poorest in Maharashtra. She was recognised for her purpose-driven zeal to work tirelessly with villagers in Maharashtra.

(f) V.Tejeswini Bai

Answer:

Tejeswini is a Kabbadi player from Karnataka who has represented India from 2005 to 2010 and captained Indian team for four years.

(g) Aishwarya Narkar

Answer:

Aishwarya Narkar is a Marathi actress who also works in the Hindi TV serials. She has received National Film Award from the  President of India.

UPSC CIVIL SERVICES MAINS EXAM SOLVED PAPER 2011

General Studies Paper – II

1. Answer any two of the following in about 250 words each:      20×2= 40
(a) List the Central Asian Republics and identify those of particular strategic and economic importance to India. Examine the opportunities and bottlenecks in enhancing relations with these countries.

Answer:
Central Asian Republics include Kazakhstan, Kyrgyzstan, Tajikistan, Turkmenistan, and Uzbekistan, Although all of them are important for India from the perspective of trade, security, energy security, civilisational link, but the ones particularly important are Kazakhstan, Turkmenistan and Tajikistan.

Kazakhstan is one of the fastest growing economies of Central Asia and a politically stable nation. India has signed civil nuclear agreement with Kazakhstan and agreed to give India a foothold in the oil rich Caspian through Satapayev block. Central Asian republics are important to India for its energy security (vast reserves of oil, gas and radioactive material), a potential market for its growing trade, strategically vital to safeguard India’s interest in Central Asia, West Asia and Africa. TAPI gas pipeline agreement signed recently involving Turkmenistan promises to address some of India’s energy needs through supply of natural gas. Central Asian Republic are also potential markets for export of Indian pharmaceuticals, IT services, entertainment services etc.

These nation are also part of regional groupings like SCO which will play crucial role in regional stability and prosperity in Asia and hence in the world.

On the flip side of it, some of these nations are not politically stable, lack vibrant democracy, and are marred by ethnic clashes which make it difficult for the businesses to operate. Lack of infrastructure and land connectivity also places obstacles in the smooth business activity. The same product that would take more than a month to reach these nations from India can be obtained within a week from China, Turkey or Europe. Terrorism emanating from the Central Asian region like from Fergana valley is another area of concern and challenge for India.
  
(b) Critically examine the security and strategic implications of the so-called ‘string of pearls’ theory for India.

Answer:
String of pearls theory refers to China building deep water ports in the Indian Ocean like Gwadar (Pakistan), Hambantota (Sri Lanka), Sittwe(Mayanmar), Chittagong (Bangladesh)  as a part of the strategy to encircle India. China denies this and claims that it is not directed against any nation. It is rather for securing its energy supplies and sea routes for its trade.

Several western analysts including some Indian Defence analyst think otherwise and claim that it is part of China’s strategy to contain India in the event of any future conflict. India is surrounded by sea from three sides and a naval dominance of China or any other nation in Indian Ocean is certainly a security concern for India. String of pearls makes China India’s maritime neighbour and give the opportunity to China to have a two front situation against India in case of a conflict.

Strategically China’s growing indulgence with India’s neighbours like Sri Lanka or Bangladesh through these ports gives China an extra lever to use against India. It can be used both as soft and a hard power to gain strategic depth inside India’s neighbourhood.
India’s rise on the world stage is guided mostly by its economic might in terms of its rising GDP and share in world trade which depends largely on the sea routes through which the trade is conducted. Any dominance on the sea routes can be used against India’s growth and thus requires India to take cognizance of the String of Pearls strategy by china.  

(c) “Compared to the South Asian Trade Area (SAFTA), the Bay of Bengal Initiative for Multisectoral Technical and Economic Cooperation Free Trade Area (BIMSTEC FTA) seems to be more promising.” Critically evaluate.

Answer:
BIMSTEC was initiated with the goal to combine the 'Look West' policy of Thailand and ASEAN with the 'Look East' policy of India and South Asia. So it could be explained that BIMSTEC is a link between ASEAN and SAARC. BIMSTEC provides a unique link between South Asia and Southeast Asia bringing together 1.3 billion people - 21 percent of the world population.

Compared to south Asian free trade area (SAFTA), the BIMSTEC FTA seems to be more promising. A deeper economic integration process within South Asian Association of Regional Cooperation (SAARC) sometime suffers because of political tension between India and Pakistan. Such things are less likely to happen in case of BIMSTEC. It is believed that negotiation under the BIMSTEC umbrella will be easier than under SAFTA because all the BIMSTEC members are purely guided by economic interests rather than by political interests. Many countries in SAFTA have less favorable geographical location in terms of being land locked and thus have adverse affect on trade as compared to BIMSTEC nations. BIMSTEC being a link between South Asia and South East Asia also opens up vast opportunities for trade as compared to SAFTA which is limited to mainly South Asia.

When compared in terms of their economic structure, namely, value addition of services, industry, and agriculture sector, to gross domestic product (GDP), BIMSTEC nations have many similarities. Except in case of Thailand, the industrial sector constitutes roughly a fourth of GDP in all countries. All these economies are predominantly associated with service related activities. Although majority of the population still lives in rural areas, all of these nations are becoming increasingly urbanised. Geographical proximity along with similar economic profile indicates complementarities in consumption, production, and trading pattern. All these factors make BIMSTEC FTA more promising.


2. Answer any three of the following in about 150 words each:     15×3=36
(a) Subsequent to the Nuclear Supplier Group (NSG) waiver in 2008, what are the agreements on nuclear energy that India has signed with different countries?

Answer:
Subsequent to NSG waiver India has signed Civil Nuclear Cooperation with nine nations and South Korea is the latest in the list. The NSG waiver ended nuclear apartheid of almost three decades for India where world declined to share any nuclear technology and resources with India even for non military purposes.

India has also incorporated enrichment and reprocessing agreements with several nations like Russia and France after the NSG waiver. Countries have guaranteed fuel supply to Indian nuclear reactors which were running much below their capacity due to lack of nuclear fuel.

India has also passed legislation on nuclear liabilities and is progressing towards ratifying Convention for Supplementary Convention which will bring India in line with the international regime and facilitate nuclear suppliers of other nations to do business with India.

(b) Trace the progress of India’s efforts for a joint counter-terrorism strategy with China. What are the likely implications of the recent Xinjiang violence on these efforts?

Answer:
India and China have completed two rounds of joint anti terrorism exercise in 2007 and 2008. Although the symbolic importance of these exercises is immense, both countries could explore cooperation in preventive measures particularly intelligence sharing; and hotline for critical situations to tackle the menace of terrorism. The two sides could consider maximizing their cooperation in the field of counter-terrorism and further consolidating strategic partnership between the two.
China’s Xinjiang problem is connected with the network of cross border and transnational terrorism. Xinjiang is a case in point of the ethno-religious separatist problem in the Xinjiang region.

Even after having such immense opportunities, India and China defense cooperation has been stalled due Chinese policy of stapled visa or denial of visa for Indian army officers from Jammu and Kashmir region and the state of Arunachal Pradesh.

(c) Bring out the importance of the Small and Medium Enterprises Expo and Conference held in Dubai last year for Indian business.
Answer:
The 2010 SME Conference theme was Meet, discuss, start. Last year’s conference was designed to provide information and assistance to all sizes of small businesses, as well as to those from the government who work with them. The conference created opportunities for small business owners and entrepreneurs to network, build alliances, and learn about new products, services and trends. Relevant educational presentations provided participants with new strategies and tools that can be put into action.

The conference was important for Indian business because of following reasons
• Opportunities for SME
• Strategies to boost the business environment for SMEs
• Investments and financing SME growth
• Banking and financial service innovations for SMEs
• Technologies that can help SME growth
• Women Entrepreneur

Small and Medium Enterprises Expo and Conference, offered great platform to small units to promote their brands, products and services, as well as to explore business opportunities in the Middle East and North African markets.

Besides giving a major impetus to networking activities with other visiting countries such as Pakistan, Mauritius, Nigeria and Vietnam, the event offered Indian SMEs the opportunity to acquaint themselves with the latest technologies in their industries and form alliances with international clients. Indian SMEs from sectors such as plastic, petrochemicals, garments and textiles, and art and handicrafts, among others gained from the participation at the expo.
(d) What are the salient features of the political and economic relationship between India and South Africa?

Answer:
South Africa has emerged as an important ally for India in recent times. Apart from having civilisational links and India’s close support during the apartheid movement and Mahatma Gandhi’s proximity with the cause South Africa is also a symbol of India’s engagement with the African people and society.

India and South Africa are part of IBSA and BRICS. IBSA represents south-south alliance while BRICS is concerned more with the economic cooperation among the member states. India and South Africa have given coordinated response in G20 and WTO. South Africa is an important destination for investment by the Inidan Business Community and a joint CEOs forum has been set up between the two nations to deepen economic cooperation and raise the level of bilateral trade.

The two nations have demanded reforms of the international institutions, given coordinated responses in the climate change negotiations with the developed nations. They coordinated and put a joint stand on the issue of Syria which provided a third alternative or a middle path in dealing with such issues of international intervention. The two nations also have a shared understanding of Iran’s nuclear issue.
    
3. Answer either of the following in about 250 words:    20
(a) “The causes and implications of the Jasmine Revolution and its spread are as much economic in nature as they are political.” Critically evaluate.

Answer:
The region of West Asia and North Africa is in a state of flux and going through a turbulent phase. The ripples that started in Tunisia from Jasmine revolution has spread far and wide. Many regimes like in Egypt and Libya have fallen since then while some have been truly shaken like in Syria, Behrain and Yemen.

The region shares some common economic and political characteristics that have resulted in this. These regimes have been more or less dictatorial in nature, giving minimum or no political liberties to its citizens. The freedom of speech, expression and movement has been restricted in past with no credible democracy. Elections were often rigged and constitutions manipulated to suit the ruling elite. The growing reach of media and social networking sites on the other hand opened the window for their people to peep into the outside free world. This raised their expectations from their own rulers and also provided a platform to organise protests against the regimes. Underlying ethnic and sect tensions against the regimes have also added fuel to the fire.

Economic reasons like high rate of unemployment among youth as in case of Tunisia, and low per capita income have also resulted in these revolts. The rulers in these nations have indulged in lavish life style and amassed treasures while the fellow citizens were starving and struggling with poor education, health and public utilities system. None of these nations had any proper higher education system which could ensure respectable jobs to its youth. Many countries like Bahrain and Saudi Arabia have distributed cash and benefits among its citizens since the start of Jasmine revolution but that does not address the systemic reforms that the people these nations are demanding.
        
(b) In the context of the ‘Euro-zone’ debt crisis, examine the proposed ‘six-pack’ solution. Do you think that this has a better chance of success than the earlier stability and growth pact?

Answer:
The six pack solution has been proposed for Euro Zone debt crisis which threatens to drag the world economy along with it into yet another full blown crisis. It is a set of legislative measures that gives European Commission the ability to impose sanctions on the eurozone countries that fail to control their debt and deficit. The rules are designed to prevent any future debt crisis of the present nature and thus give more sanctioning power to European Commission.

The European parliament will have the right to call finance ministers from countries that have been warned to hearings, while the European semester provides for the annual assessment of national budgets. Falsified debt and deficit statistics can lead to a fine of 0.2 per cent of gross domestic product. And countries will be obliged to act pre-emptively to maintain the health of economies threatened by imbalances such as a housing-bubble.

The legislation would discourage member states from evading their responsibilities to each other to ensure the stability of the eurozone. The rules would provide for enhanced monitoring and surveillance of imbalances like unemployment, credit growth and housing bubble etc. This would also give right signals to the financial markets about the seriousness of the EU and its resolve to handle any future debt crisis.

4. Answer any four of the following in about 150 words each:     12×4=48
(a) “As regards the increasing rates of melting of Arctic Sea ice, the interests of the Arctic Council nations may not coincide with those of the wider world”. Explain.

Answer:
The climate change has increased the rate of melting of the Arctic Sea Ice. This has increased the access of earlier hidden oil and gas resources. Many Arctic countries like Russia, Denmark, Canada and Iceland have rushed to claim their sovereignty over these resources. They see the climate change as an opportunity to exploit these hidden resources. They also provide new opportunities for fishing and shipping. 

Wider world on the other hand is concerned about the increasing rate of melting of Arctic Ice as this can threaten existence of some species, disturb the water and ocean cycle, adversely affect global average temperature, and can also lead to loosing climate history stored in these ice sheets. This is a ecologically sensitive area and highly unregulated thus causes concerns for the world at large.
   
(b) Is there still a role for the concept of balance of power in contemporary international politics? Discuss.

Answer:
The balance of power in the contemporary international politics can be used not only in terms of one super power balancing the other or regional balance of power but also the balance between hard and soft power. The disintegration of USSR tilted the balance of power in favour of America but economic rise of China has started to threaten the numero uno position of USA.

Several example of balance of power in international politics is seen. USA uses Israel as a tool to balance power in West Asia. NAM during the cold war era was used by developing countries to balance the relationship between two superpowers. The doctrine that neighbour’s neighbour is a friend is seen in several situations like China having all weather friendship with Pakistan to balance India. Chinese allege that USA is using its neighbours to contain the rise of China.

The growing economic prowess of emerging economies like India, China, Brazil and South Africa is being used in international forums like G20 or WTO to balance the developed nations. Thus the concept is very much alive even though it might have weakened a bit in the interconnected globalized world with supra national organisations like United Nations playing a key role in international politics.      
(c) “Strategic interests seem to be replacing commercial interests for the host country with regard to Cam Ranh bay.” Amplify.

Answer:
Vietnam opened its Cam Ranh Bay after eight years of its closure. The bay is in the northeast of Ho Chi Minh City. It is one of the best deepwater shelters in Southeast Asia. Ships can stop here for refuelling or repair. It enjoys a geo strategic significance as it is located near to key shipping lanes in the South China Sea. China’s aggressive policy in the South China Sea also led to this decision by Vietnam.

The Bay has been strategically crucial for great powers like Russia, Japan, France and USA in the history. Vietnam had not opened it after Russian withdrawal in 2002 but the recent claims by China seems to have led Vietnam to take this step. Opening of the Bay for the navies of many nations will indirectly strengthen Vietnam’s claim and right in the South China Sea and weaken Chinese position. 

(d) To what extent has the withdrawal of al-Shabab from Mogadishu given peace a real chance in Somalia? Assess.

Answer:
Al-Shabab has withdrawn from most parts of the capital of Somalia but its fighters still control some areas, including Deynile. The Islamist group withdrew after a government offensive to retake the city and clear the way for foreign aid destined for drought and famine victims. African Union troops has helped push back the rebels.

Al-Shabab once controlled nearly all of Mogadishu and still controls large swaths of central and southern Somalia. The group has tightly controlled the delivery of aid to famine victims in its territory, and has banned access for many international aid agencies. Thus its withdrawal provides real opportunity for peace but it also depends on how this opportunity is utilised by the Somalis government.

Al-Shabab on the other hand has claimed that the retreat was a strategic move and it will remain nearby and continue its effort to topple the United Nations-backed government.

(e) On a Formula-one (F-1) racing car track, information to drivers is generally signalled through the standardized use of flags of different colours. Describe the meaning associated with any six of the flags listed below:
(i) White Flag

Answer:
When the white flag is waved by the race marshals, it means the drivers should immediately slow down, as it indicates the presence of a safety car, ambulance or towing truck ahead, on the track. In this situation, overtaking is strictly prohibited.

(ii) Black Flag

Answer:
Once a race marshal waves the black flag and it attaches the race number to the car to it that means the driver is disqualified for the ongoing race. After seeing the black flag, a driver must enter the pits within the next lap and report immediately to the Clerk of the Course.

(iii) Yellow Flag

Answer:
Indicates danger ahead and overtaking is prohibited. A single waved yellow flag means slow down, a double waved yellow warns that the driver must be prepared to stop if necessary.

(iv) Blue Flag

Answer:
Shown to a driver to indicate that a faster car is behind him and trying to overtake. Shown both to lapped cars and those racing. A lapped car must allow the faster car past after seeing a maximum of three blue flags or risk being penalised.

(v) Black and White Flag divided diagonally
Answer:
Shown with car number to indicate a warning for unsportsmanlike behaviour. A black flag may follow if the driver takes no heed of the warning.

(vi) Chequered Flag

Answer:
This shows that the race has ended. Shown first to the winner, and then to every car to cross the line behind them.
(vii) Yellow and Red Striped Flag

Answer:
The track is slippery. This usually warns of oil or water on the track.

5. Comment on any thirteen of the following in about 50 words each:    5×13=65
(a) International Year of Chemistry

Answer: 
The United Nations declared 2011 to be the International Year of Chemistry. The year 2011 marks the 100th anniversary of the founding of the International Association of Chemical Societies, as well as the year Madame Marie Curie won the Nobel Prize – which celebrates the contributions of women to science. The Year hopes to promote the appreciation of chemistry in meeting world needs and the future development of chemistry.

(b) The scourge of e-waste

Answer:
e-waste constitutes of used, obsolete, and end of life cycle electronics product which if disposed in an unscientific manner can lead to pollution of air, water and soil. Acid bath, land fill and burning are used for the disposal of e-waste leading to pollution through heavy metals (like cadmium, mercury, and nickel) poly vinyl chloride and poly chlorinated biphenyl. e-waste is from developed countries is being dumped to developing countries causing large scale health hazards.       
(c) Designer poultry eggs
Answer:
Designer eggs are for beauty conscious consumers, as well as persons affected with diabetes and heart-disease. It will be bacteria-free, rich in protein and high on calorific value. The designer eggs are produced through a scientific method adopted for nurturing the layer with the right feed so that they lay high quality eggs. The composition of the feed leads to the desired composition of the eggs.

(d) INSPIRE programme of the Department of Science and Technology

Answer:
Innovation in Science Pursuit for Inspired Research (INSPIRE)" is an innovative programme sponsored and managed by the Department of Science & Technology for attraction of talent to Science. The basic objective of INSPIRE is to communicate to the youth of the country the excitements of creative pursuit of science, attract talent to the study of science at an early age and thus build the required critical human resource pool for strengthening and expanding the Science & Technology system and R&D base.

(e) The Kessler syndrome with reference to space debris
Answer:
The Kessler Syndrome is a future scenario when the amount of space junk reaches a high enough density that each collision will produce sufficient fragments that generate a slow cascade effect; producing more collisions and debris, eventually causing our species to become incapable of launching space craft. It was first proposed in 1978 by Donald J. Kessler when debris that accumulates in space is increasing faster than what debris falls out of orbit and burns up.

(f) Omega-3 fatty acids in our food

Answer:
Omega-3 fatty acids are polyunsaturated fats found naturally in oily fish, nuts, seeds, and leafy green vegetables. Omega-3 fatty acids are thought to protect against heart disease, inflammation, and certain types of cancer, diabetes, Alzheimer’s disease, and macular degeneration (a leading cause of vision loss). Omega-3 fatty acids are critical for proper brain development and neurological function in developing babies, too.

(g) Difference between ‘spin-drying’ and ‘tumble-drying’ technology with reference to drying of washed clothes

Answer:
Spin drying technology for drying washed clothes saves time as the high speed and RPM (revolutions per minute) allows the clothes to get rid of water quickly due to centrifugal force. Tumble drying technology on the other hand works mainly on the heated air being passed through the clothes to dry them up. Heat and not spinning speed is used to dry the clothes through this technology.

(h) The admonishing population of vultures

Answer:
The vulture population has been declining which was a cause of concern. Use of diclofenac with the cattle population is partly responsible for this decline. Bombay Natural History Society has taken up conservation and awareness efforts which have positively impacted the vulture population. Vulture breeding centres are set up in Pinjore(Haryana), Rani forest (Assam) to arrest the declining population. The slender billed, white backed vultures are among the endangered species.

(i) ‘Arsenic-bug’ and the significance of its discovery

Answer:
NASA-supported researchers have discovered the first known microorganism on Earth able to thrive and reproduce using the toxic chemical arsenic. The microorganism, which lives in California's Mono Lake, substitutes arsenic for phosphorus in the backbone of its DNA and other cellular components. Carbon, hydrogen, nitrogen, oxygen, phosphorus and sulfur are the six basic building blocks of all known forms of life on Earth. This finding will alter biology textbooks and expand the scope of the search for life beyond Earth.

(j) F-22 ‘Raptor’ aircraft

Answer:
Developed by Lockheed Martin/Boeing, the F-22A Raptor is a supersonic, dual-engine fighter jet. The F-22 is designed for stealth, supercruise speed and super-agility.

It is a fifth generation aircraft. . It has additional capabilities that include ground attack, electronic warfare, and signals intelligence roles. It lost the Indian bid to Eurofighter and French Rafale recently.

(k) ‘Concentrated’ solar energy and’ photovoltaic’ solar energy

Answer:
Concentrating solar power (CSP) technologies use mirrors to reflect and concentrate sunlight onto receivers that collect the solar energy and convert it to heat. This thermal energy can then be used to produce electricity via a steam turbine or heat engine driving a generator.Photovoltaic solar energy is method of converting solar energy into electrical energy using photovoltaic cells having semiconductors that exhibit photoelectric effects. Photovoltaic cells could use any of the semiconductors like silicon, selenium, gallium.

(l) Analog hybrid and IP systems in CCTV technology
Answer:
Monitoring and surveillance applications were traditionally done by analog CCTV technology.  Analog CCTV systems are generally maintenance intensive, offer no remote accessibility, and are notoriously difficult to integrate with other systems. IP systems in CCTV technology gets rid of these obstacles.  It provides ease of use, advanced search capabilities, simultaneous record and playback, no image degradation, improved compression and storage, and integration potential. IP CCTV technology allows us to take advantage of  many new technologies like LAN, Broadband, VPN etc.

(m) Various applications of Kevlar

Answer:
Originally developed as a replacement for steel in radial tires, Kevlar is now used in a wide range of applications.  The Kevlar tire technology has been applied to aeroplane, car, racing vehicle and truck tires. Some components of Kevlar are used in an array of rackets, such as tennis, badminton and squash rackets. Canoes and kayaks were also improved when Kevlar technology was applied there.

(n) Differences between Compact Disc (CD), Digital Versatile Disc (DVD) and Blu-ray Disc.
Answer:
The Compact Disc is an optical disc used to store digital data. A DVD is an optical disc storage media format, developed by Philips, Sony, Toshiba, and Panasonic. DVDs offer higher storage capacity than Compact Discs while having the same dimensions.  Blu-ray Disc is an optical disc storage medium designed to supersede the DVD format. The plastic disc is 120 mm in diameter and 1.2 mm thick, the same size as DVDs and CDs. Blu-ray Discs contain 25 GB per layer. Blue ray gets its name from the blue laser used to read the data. It allows information to be stored at a greater density as compared to red laser being used in DVD.

6. Comment on the following in about 50 words each:    5×5=25

(a) Functions of the World Customs Organisation (WCO)

Answer:
The World Customs Organization is the only intergovernmental organisation exclusively focused on Customs matters. WCO is recognised as the voice of the global Customs community. It works in the areas covering the development of global standards, the simplification and harmonisation of Customs procedures, trade supply chain security, the facilitation of international trade, the enhancement of Customs enforcement and compliance activities, anti-counterfeiting and piracy initiatives, public-private partnerships, integrity promotion, and sustainable global Customs capacity building programmes.

(b) Success of international intervention of Cote d’Ivoire (Ivory Coast)

Answer:
UN Security council had authorised the intervention in Cote d’Ivoire to end the standoff created by the Gbagbo who had refused to give power after defeat in Presidential elections.  UN peacekeeping French forces intervened and the intervention in Côte d'Ivoire worked. Within a week, former president Laurent Gbagbo, who had refused to accept defeat in an election and plunged his country into a steadily escalating spiral of violence and repression, was in custody, and within two weeks the majority of his forces had surrendered or rallied to the new President's side.

(c) Strategic adopted by Colombia to eliminate its drug cartels
Answer:
Columbia has adopted a comprehensive strategy to eliminate drug cartels which involves reducing or eliminating corruption from the police force, judiciary and all the drug enforcement agencies. Several operations involving undercover agents were also undertaken to expose the financial network used by the drug mafia for money laundering. Columbia is also making international efforts to have a coordinated response to the menace of drug trafficking.

(d) World Food Programme (WFP) of the United Nations (UN)

Answer:
WFP is the United Nations frontline agency in the fight against global hunger. WFP is the food aid arm of the United Nations system. Food aid is one of the many instruments that can help to promote food security. The policies governing the use of World Food Programme food aid is oriented towards the objective of eradicating hunger and poverty. The ultimate objective of food aid is the elimination of the need for food aid.

(e) Sculpture of the broken chair in front of the UN building at Geneva

Answer:
Sculpture of the broken chair symbolises opposition to land mines and cluster bombs, and acts as a reminder to politicians and others visiting Geneva. The sculpture was erected by Handicap International and is a work by the Swiss artist Daniel Berset.

7. Why each of the following been in the news recently? (each answer in a sentence or two only);     2×10=20

(a) Tiangong – 1

Answer:
It is a Chinese space laboratory module,and is an experimental testbed to demonstrate the rendezvous and docking capabilities needed to support a space station complex.

(b) K-computer

Answer:
It is a supercomputer produced by Fujitsu at the RIKEN Advanced Institute for Computational Science campus in Kobe, Japan. The K computer is based on distributed memory architecture.

(c) Gliese 581 g

Answer:
It is a hypothetical extrasolar planet of the red dwarf star Gliese 581, in the constellation of Libra. It is the sixth planet discovered in the Gliese 581 planetary system and the fourth in order of increasing distance from the star.

(d) MABEL robot

Answer:
It is believed to be the world’s fastest bipedal robot with knees. It is built with funding from the National Science Foundation and the Defence Advanced Research Projects Agency, USA.

(e) Operation Shady Rat

Answer:
It is referred to the series of cyber attacks starting from mid 2006. It is characterized by McAfee as a five year targeted operation by one specific actor. It refers to targeting of several athletic organisations around the time of Summer 2008 Olympics.

(f) SAGA-220

Answer:
It is the fastest super computer of India developed by ISRO. Its speed is 220 teraflops.

(g) Billion Acts of Green

Answer:
The goal of Billion Acts of Green campaign is to reach a billion acts of environmental service and advocacy before Rio +20 to be held in 2012.

(h) L’Aquila earthquake
Answer:
Six scientists have been accused for manslaughter over the 2009 L’Aquila earthquake. Prosecutors allege the defendants gave a falsely reassuring statement before the quake after studying hundreds of tremors that had shaken the city.
(i) OPERA detector at Gran Sasso
Answer:
Oscillation Project with Emulsion-Racking Apparatus is an experiment to detect neutrinos. OPERA researchers have observed muon neutrinos travelling apparently at faster than the speed of light.

(j) Saturn’s Titan

Answer:
Titan is the largest moon of Saturn and the second largest moon in the solar system. It is the only natural satellite known to have a dense atmosphere.

8. Why have the following been in the news recently? (each answer in a sentence or two only):     2×5=10

(a) ‘News International’ newspaper

Answer:
News International is the UK arm of Murdoch's global News Corp empire and publishes four British newspapers: the Times, the Sunday Times, the Sun and the News of the World, as well as the weekly Times Literary Supplement.

(b) Mustafa Abdul-Jalil

Answer:
He is the Chairman of the National Transitional Council of Libya, and as such serves as head of state in Libya's caretaker government which was formed as a result of the 2011 Libyan civil war.

(c) Abel Kirui

Answer:
Kenya's Abel Kirui is a marathon runner who retained his world championships title in emphatic fashion, recording the largest winning margin in championship history.

(d) Natalie Portman
Answer:
She is a hollywood actress who has won the Oscar for the best actress for her performance in the movie Black Swan.

(e) Nawaf Salam

Answer:
He is a Lebanese diplomat, academic, and jurist. He is currently serving as Lebanon's Ambassador and Permanent Representative to the United Nations in New York.

9. (a) List the requisites of a good table:
Present the following in a suitable tabular form:     5

In 1980 out of a total of 1750 workers of a factory, 1200 were members of a union. The number of women employed was 200 of which 175 did not belong to the union. In 1985 the number of union workers increased to 1580 of which 1290 were men. On the other hand the number of non-union workers fell down to 208 of which 180 were men. In 1990 there were 1800 employees who belonged to the union and 50 who did not belong to the union. Of all the employees in 1990, 300 were women of which only 8 did not belong to the union.
Answer:

Male Female

Union Non Union Union Non Union
1980 1175 375 25 175
1985 1290 180 290 28
1990 1508 42 292 8
(b) Draw an ogive for the following distribution. Read the median from the graph. How many students get between 60 and 72?

Marks No. Of Students
50-55 6
55-60 10
60-65 22
65-70 30
70-75 16
75-80 12
80-100 15
(c) From the following data calculate the missing frequency:    4

No. of tablets No. of Persons cured
4-8 11
8-12 13
12-16 16
16-20 14
20-24 ?
24-28 9
28-32 17
32-36 6
36-40 4
The average number of tablets to cure fever was 19.9
Answer:

No. of tablets Middle Point No. of Persons cured Product
4-8 6 11 66
8-12 10 13 130
12-16 14 16 224
16-20 18 14 252
20-24 22 x 22x
24-28 26 9 234
28-32 30 17 510
32-36 34 6 204
36-40 38 4 152

1772+22x/90+x = 19.9
1772 +22x =  1791 + 19.9x
2.1x = 19
x = 9
(d) Life-time of 400 tubes tested in a company is distributed as follows:

Life time (hours) No. of tubes
300-399 14
400-499 46
500-599 58
600-699 76
700-799 68
800-899 62
900-999 48
1000-1099 22
1100-1199 6
Determine :     4
(i)
Relative frequency of sixth class.
Answer: See Answer table below
Relative frequency of sixth class is = Frequency of that class/ Total frequency = 62/ 400 = 0.155 = 15.5%
(ii) Percentage of tubes whose life-time does not exceed 600 hours.
Answer:Percentage of tubes whose life-time does not exceed 600 hours = 29.5% from table below

(iii) Percentage of tubes whose life time is greater than or equal to 900 hours.
Answer:Percentage of tubes whose life time is greater than or equal to 900 hours = 19% from table below

(iv)
Percentage of tubes whose life time is at least 500 but less than 1000 hours.
Answer:Percentage of tubes whose life time is at least 500 but less than 1000 hours = (Less than 1000 hours – Less than 500 hours) / 400 = (372 – 60)/ 400 = 312/400 = 78%
 
Life time Less than Cumulative Fr Percentage More than Cumulative Fr Percentage No. of tubes
299.5-399.5 399.5 14 3.50% 299.5 400.00 100.00% 14
399.5-499.5 499.5 60.00 15.00% 399.5 386.00 96.50% 46
499.5-599.5 599.5 118.00 29.50% 499.5 340.00 85.00% 58
599.5-699.5 699.5 194.00 48.50% 599.5 282.00 70.50% 76
699.5-799.5 799.5 262.00 65.50% 699.5 206.00 51.50% 68
799.5-899.5 899.5 324.00 81.00% 799.5 138.00 34.50% 62
899.5-999.5 999.5 372.00 93.00% 899.5 76.00 19.00% 48
999.5-1099.5 1099.5 394.00 98.50% 999.5 28.00 7.00% 22
1099.5-1199.5 1199.5 400.00 100.00% 1099.50 6.00 1.50% 6

10. (a) A car travels 25 km at 25 kph, 25 km at 50 kph, and 25 km at 75 kph. Find the average speed of the car for the entire journey.
Answer:

Distance in km Speed in kph Time taken  in hr = Distance/ Speed

25 25 1.00

25 50 0.50

25 75 0.33
Total 75
1.83
Average Speed in kph Total Distance/ Total Time 40.91
(b) The mean of 200 items was 50. Later on it was found that two items were wrongly read as 92 and 8 instead of 192 and 88. Find the correct mean.
Answer:
Incorrect Total = 200*50 = 10000
Correct Total = 10000 + (192-92) + (88-8) = 10180
Correct Mean = 10180/ 200 = 50.9
(c) Students were asked how long it took them to walk to school on a particular morning. A cumulative frequency distribution was formed.
Time taken (minutes) c.f.
<5 28
<10 45
<15 81
<20 143
<25 280
<30 349
<35 374
<40 395
<45 400
(i) Draw a cumulative frequency curve.
(ii) Estimate how many students took less than 18 minutes.
(ii) 6% of students took x minutes or longer. Find x.    6

(d) An investor buys ` 1200 worth of shares in a company each month. During the first five months he bought the shares at a price of ` 10, ` 12, ` 15, ` 20 and ` 24 per share. After 5 months, what is the average price paid for the shares by him?    4
Answer:
Number of shares in first month = 1200/10 = 120
Number of shares in second month = 1200/12 = 100
Number of shares in third month = 1200/15 = 80
Number of shares in fourth month = 1200/20 = 60
Number of shares in fifth month = 1200/24 = 50
Total number of shares = 410
Total money spent = 1200*5 = 6000
Average price for shares = 6000/ 410 = 14.6

Friday, January 6, 2012

INDIAN POLITY STUDY MATERIAL

 

The Republic of India is governed in terms of the Constitution, which was adopted by Constituent Assembly on 26 November 1949 and came-into force on 26 January 1950.
  • The President of India is constitutional head of executive of the Union.
  • Article 74(1) of the Constitution provides that there shall —a Council of Minister with the Prime Minister as head to aid and advise President who shall in exercise of his functions act in accordance with such advice.
  • Real executive power thus vests in Council of Ministers with Prime Minister as head.
  • Council of Ministers is collectively responsible to the House of the People (Lok Sabha).
  • Similarly, in states, Governor is head of executive, but it is the Council of Ministers with Chief Minister as head in whom real executive power vests.
  • Council of Ministers of a state is collectively responsible to the Legislative Assembly.
  • The Constitution distributes legislative power between Parliament and state-legislatures and provides for vesting of residual powers in Parliament.

Citizenship

  • The Constitution of India provides for a single citizenship for the whole of India.
  • Acquisition of Citizenship: The Citizenship Act, 1955 prescribes five ways of acquiring citizenship.
  1. By Birth: If he is born in India on or after 26 January, 1950 but before 30 June, 1987; or if he is born in India on or after 1 July 1987 but at the time of his birth either of his parents was a citizen of India.
  2. By Descent: A person born outside India on or after 26 January, 1950, is a citizen of India by descent if at the time of his birth either of his parents was an Indian Citizen. But before person born outside India was entitled to Indian Citizenship only if his father (and not his mother) was an Indian Citizen.
  3. By Registration
  4. By Naturalisation: A foreigner can become an Indian citizen by naturalisation.
  5. By Incorporation of Territory

Loss of Citizenship

  • By Renunciation: An Indian citizens who is also a citizen or national of another country, can renounce his Indian. Citizenship.
  • By Termination: When an Indian Citizen voluntarily acquires the citizenship of another country, his Indian citizenship automatically terminates.
  • By Deprivation: It is a compulsory termination of Indian citizenship by the Central Government.

FUNDAMENTAL RIGHTS

  • These are guaranteed in the Constitution in the form of six broad categories of Fundamental Right which are justiciable.
  • Article 12 to 35 contained in Part III of the Constitution deal with Fundamental Rights.

Fundamental Rights

Category Provisions
1. Right to equality (Articles 14-18) (a) Equality before law and equal protection of laws (Article 14).
(b) Prohibition of discrimination on grounds of religion, race, caste, sex or place of birth (Article 15).
(c) Equality of opportunity in matters of public employment (Article 16).
(d) Abolition of untouchability (Article 17).
(e) Abolition of titles except military and academic (Article 18).
2. Right to freedom (Article 19-22) (a) Protection of six rights in respect of: (i) speech and expression,
(ii) assembly, (iii) association, (iv) movement, (v) residence, and (vi) profession (Article 19).
(b) Protection in respect of conviction for offences (Article 20).
(c) Protection of life and personal liberty Article 21).
(d) Right to elementary education (Article 2 IA).
(e) Protection against arrest and detention in certain cases (Article 22).
3. Right against exploitation (a) Prohibition of traffic in human beings and forced labour (Article 23-24) (Article 23).
(b) Prohibition of employment of children in factories, etc. (Article 24)
4. Right to freedom of religion (a) (Article 25-28) Freedom of conscience and free profession, practice and propagation of religion (Article 25)
(b) Freedom to manage religious affairs (Article 26)
(c) Freedom from payment of taxes for promotion of any religion (Article 27)
(d) Freedom from attending religious instruction or worship in certain educational institutions (Article 28).
5. Cultural and educational rights (Articles 29-30) (a) Protection of language, script and culture of minorities (Article 29).
(b) Right ofminorities to establish and administer educational institutions (Article 30).
6. Right to constitutional remedies (Article 32) Right to move the Supreme Court for the enforcement of fundamental rights including the writs of (i) habeas corpus (ii) mandamus, (iii)
prohibition, (iv) certiorari, and (v) quo warrento (Article 32)

Fundamental Duties

  • By the 42nd Amendment of the Constitution, adopted in 1976, Fundamental Duties of the citizens have also been enumerated.
  • Article 51 ‘A’ contained in Part IV A of the Constitution deals with Fundamental Duties.
  • According toArticle.51A, it shall be the duty of every citizen of India.
(a) to abide by the Constitution and respect its ideals and institutions, the National Flag and the National Anthem;
(b) to cherish and follow the noble ideals which inspired our national struggle for freedom;
(c) to uphold and protect the sovereignty, unity and integrity of India;
(d) to defend the country and render national service when called upon to do so;
(e) to promote harmony and the spirit of common brotherhood amongst all the people of India transcending religious, linguistic and regional or sectional diversities; to renounce practices derogatory to the dignity of women;
(f) to value and preserve the rich heritage of our composite culture;
(g) to protect and improve the natural environment including forests, lakes, rivers and wild life, and to have compassion for living creatures;
(h) to develop the scientific temper, humanism and the spirit of inquiry and reform;
(i) to safeguard public property and to abjure violence;
(j) to strive towards excellence in all spheres of individual and collective activity so that the nation constantly rises to higher levels of endeavour and achievement.
(k) to provide opportunities for education to his child or ward between the age of six and fourteen years (This duty was added by the 86th Constitutional Amendment Act 2002)

Directive Principles of State Policy

  • The Constitution lays down certain Directive Principles of State Policy which though not justiciable, are ‘fundamental in governance of the country’ and it is the duty of the State to apply these principles in making laws.

Gandhian Principles

  • Article 40: State shall organize village panchayat as unit of self-government.
  • Article 43: State shall try to promote cottage industries.
  • Article 46: State shall promote with special care the educational and economic interests at the weaker sections of the peoples: SC and ST.
  • Article 47 State shall try to secure the improvement of public -health and the prohibition of intoxicating drinks and drugs.
  • Article 48: State shall preserve and improve the breeds and prohibit the slaughter of cows, calves and other cows and drought cattle.

Directives Inspired by Western Liberal Ideas

  • Article 44 : to secure a uniform civil code throughout the territory of India.
  • Article 45: Free compulsory and primary education for the children upto 14 years.
  • Article 49 : to-protect and maintain places of - historic or a rustic interests.
  • Article 50 : to work towards separating the judiciary from the executive.
  • Article 51: to. promote international peace and amity,maintain honourable relations between nations, foster respect for international law and treaty obligation.

Directives Inspired by Socialistic Ideas

  • Article 38: To promote the welfare of the people and to secure a just social order.
  • Article 39: (1) To secure adequate livelihood for all citizens; (2) distribution of ownership and control of material resources of the community to serve common good; (3) an economic system which does not lead to concentration of wealth; (4) equal pay for equal work for both men and women; (5) health and strength of worker; (6) opportunity and facilities for children to develop in a healthy manner and be protected fromexploitation.
  • Article 39A: Equal Justice and free legal aid.
  • Article 41: To make effective provision for securing the right to work, right to education, right to public assistance in case of unemployment, old age sickness and disability
  • Article 42: Asks for provision for just and human condition of work, and for maternity relief.
  • Article 43: Living wage, etc., for workers.
  • Article 43A: Participation of workers in management of industries.
  • Article 45: Free and compulsory education for children upto 14 years.

Ideals of Social Security

  • Right to work, to education and public assistance in certain cases (article 41). Recently the Supreme Court declared, in Mohini Jam’s Case that the right to education be equated with a fundamental right and it should be read with Article 22.This is because the right to life means a dignified life, which has nomeaning without education.
  • Free and compulsory education for children upto 14 years. (Article 45)
  • Promotion of educational and economic interests of the weaker sections. (Article 46)
  • Raising the standard of living and improvement of health. (Article 47)
  • Equal justice and free legal aid. (Article 39 A).
  • Just and human conditions of work. (Article 42)

The Union Executive

  • The Union executive consists of the President, the Vice-President, Attorney General and the Council of Ministers with the Prime Minister as the -head to aid and advise the President.

President

  • The President is elected by members of an electoral college consisting of elected members of both Houses of Parliament and Legislative Assemblies of the states in accordance with the system of proportional representation by means of single transferable vote. The President must be a citizen of India, not less than 35 years of age and qualified for election as member of the Lok Sabha.
  • His term of office is five years and he is eligible for re-election.
  • His removal from office is to be in accordance with procedure prescribed in Article 61 of the Constitution.
  • He may, by writing under his hand addressed to the Vice-President, resign his office.
  • Executive power of the Union is vested in the President and is-exercised by him either.

Articles Related to President

Article No. Subject-matter
Article 52 The President of India
Article 53 Executive Power of the Union
Article 54 Election of President
Article 55 Manner of election of President -
Article 56 Term of office of President
Article 57 Eligibility for re-election
Article 58 Qualifications for election as President
Article 59 Conditions of President's office
Article 60 Oath or affirmation by the President
Article 61 Procedure for impeachment of the President
Article 62 Time of holding election to fill vacancy in the office of President
Article 65 Vice-President to act as President or to discharge his functions
Article 71 Matters relating to the election of President
Article 72 Power of President to grant pardons etc. and to suspend, remit or commute sentences in certain cases.
Article 74 Council of Ministers to aid and advise the President.
Article 75 Other provisions as to ministers like appointment, term, salaries, etc.
Article 76 Attorney-General of India
Article 77 Conduct of business of the Government of India
Article 78 Duties of Prime Minister in respect to furnishing of information to the president, etc.
Article 85 Sessions of Parliament, propogation and discussion
Article 111 Assent to bills passed by the Parliament
Article 112 Union budget (annual financial statement)
Article 123 Power of President to promulgate ordinances
Article 143
  • Power of President to consult Supreme Court. directly or through officers subordinate to him in accordance with the Constitution.
  • Supreme command of defence forces of the Union also vests in him.

Vice-President

  • The Vice-President is-e1ected by members of an electoral college consisting of members of both House of Parliament in -accordance with the system of-proportional representation by means of single transferable vote.
  • He must be a citizen of India, not Less than 35 year of age and eligible for election as a member of the Rajya Sabha.
  • His term of office is five years and he is eligible re-e1ection.
  • His removal from. office is to be in accordance with procedure prescribed in Article 67 b.
  • The Vice-President is ex-officio Chairman of the Rajya Sabha and acts as President when the latter is unable to discharge his functions due to absence, illness or any other cause or till the election of a new President (to be held within six months when a vacancy is caused by death, resignation removal or otherwise of President).

Council of Ministers

  • There is a Council of Ministers headed by the Prime Minister to aid and advise the President in exercise of his functions.
  • The Prime Minister is appointed by the President who also appoints otherministers on the advice of Prime Minister.
  • The Council is collectively responsible to the Lok Sabha.
  • It is the duty of the Prime Minister to communicate to the President all decisions of Council of Ministers relating to administration of affairs of the Union and proposals for legislation and information relating to them.
  • The Council of Ministers comprises Ministers who are members of Cabinet, Ministers of State (independent charge), Ministers of State and Deputy Ministers.

Legislature

  • Legislature of the Union which is called Parliament , consists of President and two Houses, known as Council of States (Rajya Sabha) and House of the People (Lok Sabha).
  • Each’ House has to meet within six months of its previous sitting.

Rajya Sabha

  • The Constitution provides that the Rajya Sabha shall consist of-I2memberstofr,naminated by the President fromamongst person shaving special knowledge or practical experience in respect of such matters as literature, science, art and social service; and not more than 238 representatives of the States and of the Union Territories.
  • Elections to the Rajya Sabha are indirect; members representing States are elected by elected members of Legislative assemblies of the States, in accordance with the system of proportional representation by means of the single transferable vote, and those representing Union Territories are chosen in such manner as Parliament may by law prescribe.
  • The Rajya Sabha is not subject to dissolution; one-third of its members retire every second year. Rajya Sabha, at present has 244 seats.
  • Of these 233 members represent the States and the Union Territories and 11 members are nominated by the President and one seat under nomination category is vacant.

Comparison between Council of Ministers and Cabinet

Council of Ministers Cabinet
1. It is a wider body consisting of 60 to 70 ministers It is a smaller body consisting of 15 to 20 ministers.
2. It includes all the three categories of Ministers, that is, cabinet Ministers, Ministers-of state, and-deputy Ministers 2. It includes the cabinet ministers only.
3. It does not meet, as a body, to transact government business It has no collective functions 3. It as a body, frequently and usually in a to deliberate and take decisions regarding the transaction of government business. Thus, it has collective functions.
4. It is vested with all powers but in theory 4. It exercises, in practice, the powers of the council of ministers and acts for the latter.
5. It is a constitutional body, dealt in detail by the Articles 74 and 75 of the Constitution 5. It was inserted in 352 of the Constitution in 1978 by the 44th Constitution a Amendment Act
6. It is collectively responsible to the Lower House of the Parliament 6. It enforces the responsibility of the council of ministers to the Lower House of Parliament.

Lok Sabha

  • The Lok Sabha is composed of representatives of people chosen by direct election on the basis of adult suffrage.
  • The maximum strength of the House envisaged by the Constitution is now 552 (530 members to represent the States., 20members to represent the Union Territories and not more than twomembers of the Anglo-Indian community to be nominated by the President, if, in his opinion, that community is not adequately represented in the House).
  • The Lok Sabha at present consists of 545 members.
  • Of these, 530 members are directly elected from the States and 13 from Union Territories while two are nominated by the President to represent the Anglo-Indian community.
  • Following the Constitution 84th Amendment Act, 2001 the total number allocated to various States in the Lok Sabha on the basis of the 1971 census shall remain unaltered to be taken after the year 2026.
  • The term of the Lok Sabha, unless dissolved earlier is five years from the date appointed for its first meeting.
  • However, while a Proclamation of Emergency is in operation, this periodmay be extended by Parliament by law for a period not exceeding one year at a time and not extending in any case, beyond a period ofsix months after Proclamation has ceased to operate.
  • Fifteen Lok Sabhas have been constituted so far.

Comparison of Position of Rajya Sabha with respect to Lok Sabha

Rajya Sabha has equal status with Lok Sabha in following matters:
  1. Introduction and passage of’ ordinary bills.
  2. Introduction and passage of Constitutional amendment
  3. Introduction and passage of financial bills involving expenditure fromthe Consolidated Fund of India-
  4. Election and impeachment of the president.
  5. Election and removal of the vice-president.
  6. Making recommendation to the President for the removal of Chief Justice and Judges of Supreme Court and High Courts, Chief Election Commissioner and comptroller and auditor general.
  7. Approval of ordinances issued by the President.’
  8. Approval of proclamation of all three types of emergencies by the President.
  9. Selection ofministers including the Prime Minister. Under the Constitution, the ministers including the Prime Minister can be members of either House.
  10. Consideration of the reports of the constitutional bodies like Finance Commission, Union Public Service Commission, Comptroller and Auditor, etc.
  11. Enlargement of the judiciary of the Supreme Court and the Union Public Service Commission.
Rajya Sabha has unequal status with Lok Sabha in following matters:
  1. A Money Bill can be introduced only in the Lok Sabha and not in the Rajya Sabha.
  2. Rajya Sabha cannot amend or reject aMoney Bill. It should return the bill to the Lok Sabha within 14 days, either with recommendations or without recommendations.
  3. The Lok Sabha can either accept or reject all or any of the recommendations of the Rajya Sabha. In both the cases, the money bill is deemed to have been passed by the two Houses.
  4. A financial bill, not containing solely the matters of Article 110, also can be introduced only in the Lok Sabha and not in the Rajya Sabha.
  5. The final power to decide whether a particular bill is a Money Bill or not is vested in the Speaker of the Lok Sabha.
  6. The Speaker of Lok Sabha presides over the joint sitting of both the Houses.
  7. The Lok Sabha with greater number wins the battle in a joint sitting except when the combined strength of the ruling party in both the Houses is less than that of the opposition parties.
  8. Rajya Sabha can only discuss the budget but cannot vote on the demands for grants (which is the exclusive privilege of the Lok Sabha).
  9. A resolution for the discontinuance of the national emergency can be passed only by the Lok Sabha and not by the Rajya Sabha.
  10. The Rajya Sabha cannot remove the council of ministers by passing a no-confidence motion.

Special Powers of Rajya Sabha

  • It can authorise the Parliament to make a law on a subject enumeraçi1tbeSt8te List (Article 249).
  • It can authorise the Parliament to create new—All-India Services common to both the—Centre and States (Article 312).

Membership of Parliament

Qualifications
  • He must be a citizen of India.
  • He must make and subscribe before the person authorized by the election commission an oath or affirmation according to the form prescribed in the Third Schedule.
  • He must be not less than 30 years of age in the case of the Rajya Sabha and not less than 25 years of age in the case of the Lok Sabha.
  • He must possess other qualifications prescribed by Parliament.
  • He must be registered as an elector for a parliamentary constituency in the concerned state or union territory in the case of the Rajya Sabha.
  • He must be registered as an elector for some parliamentary constituency in India in the Case of the Lok Sabha.
  • He must be a member of a scheduled caste or scheduled tribe in any state or union territory, if he  wants to contest a seat reserved for them. However, a member of scheduled castes or scheduled tribes can also contest a seat not reserved for them.
Disqualifications
  • If-he holds any office of profit under the Union or State government.
  • If he is of unsound mind and stands so declare by a court.
  • If he is an undischarged insolvent.
  • If he is not a citizen of India or has voluntarily acquired the citizenship of a foreign state or is under any acknowledgement or allegiance to a foreign state;
  • If he is so disqualified under any law made by Parliament.
    Parliament laid down the following additional disqualifications in the Representation of People Act (1951).
  • He must not have been found guilty of certain election offences or corrupt practices in the elections.
  • He must not have been convicted for any offence resulting in imprisonment for two or more years. But, the detention of a person under a preventive detention law is not a disqualification.
  • He must not have failed to lodge an account of his election expenses within the time.
  • He must have any interest- in government contracts, works or services.
  • He must not be a director or managing—agent nor hold an office of profit in a corporation in which the government has at least 25 per cent share.
  • He must not have been dismissed fromgovernment services for Corruption or disloyalty to the State.
  • He must not have been convicted for promoting enmity between- different groups or for the. offence of bribery.

Parliamentary Committees

  • Their appointment, terms of office, functions and procedure of conducting business are also more or less similar and are regulated as per rules made by the two Houses under Article 118(1) of the Constitution.
  • Broadly, Parliamentary Committees are of two kinds—Standing Committees and ad hoc Committees.
  • The former are elected or appointed every year or periodically and their work goes on, more or less, on a continuous basis.
  • The latter are appointed on an ad hoc basis as need arises and they cease to exist as soon as they complete the task assigned to them.
  • Among the Standing Committees the three Financial Committees— Committees on Estimates, Public Accounts Committee and Public Undertakings constitute a distinct group as they keep an unremitting vigil over Government expenditure and performance.
  • While members of the Rajya Sabha are associated with Committees on Public Accounts and Public Undertakings, the members of the Committee on Estimates are drawn entirely from the Lok Sabha.
  • Estimates Committee: The Estimates Committee reports on ‘what economies, improvements in Organisation, efficiency or administrative reform consistent with policy underlying the estimates’ may be effected.
  • The origin of this committee can be traced to the Standing Financial Committee setup in 1921. First Estimates Committee in the post-independence era was constituted in 1950 on the recommendation of John Mathai, the then Finance Minister.
  • It also examines whether the money iswell laid out within limits of the policy implied in the estimates and suggests the form in which estimates shall be presented to Parliament.
  • Public Accounts Committee: The Public Accounts Committee scrutinises appropriation and finance accounts of Government and reports of the Comptroller and Auditor-General.
  • It ensures that public money is spent in accordance with Parliament’s decision and calls attention to cases of waste, extravagance, loss or nugatory expenditure.
  • The committee was first setup in 1921, at present it consists of 22 members (l5fromLokSabha and 7 from Rajya Sabha). The Members are elected by the parliament every year by proportional representation by means of single transferable vote. The term of the office of members is one year.
  • A minister cannot be elected as amember of committee. The chairman of the committee is appointed by the Speaker.
  • Committee on Public Undertakings: The Committee on Public Undertakings examines reports of the Comptroller and Auditor-General, if any. It also examines whether public undertakings are being run efficiently and managed in accordance with sound business principles and-prudent commercial practices.
  • The Committee was created in 1964 on the recommendation of the Krishna Menon Committee. Originally it had 15members (l0 from the Lok Sabha and 5 from the Rajya Sabha).
  • Besides these three Financial Committees the Rules Committee of the Lok Sabha recommended setting-up of 17 Department Related Standing Committees (DRSCs).
  • Accordingly, 17 department Related Standing Committees were set upon 8 April 1993.
  • In July 2004, rules were amended to provide for the constitution of seven more such committees, thus raising the number of DRSCs from 17 to 24.

Standing Committees of Parliament and their functions.

  • Committee to Inquire:
(a) Committee on Petitions examines petitions on bills and on matters of general public interest and also entertains representations on matters concerning subjects in the Union List; and
(b) Committee of Privileges examines any question of privilege referred to it by the House or Speaker! Chairman;
  • Committees to Scrutinise:
(a) Committee on Government Assurances keeps track of all the assurances, promises, undertakings, etc.
(b) Committee on Subordinate Legislation scrutinises and reports to the House whether the power to make regulations, rules, sub-rules, bye-laws, etc.
(c) Committee on Papers Laid on the Table examines all papers laid on the table of the House by Ministers, other than statutory notifications and orders which come within the purview of the Committee on Subordinate Legislation, to see whether there has been compliance with the provisions ôf1te Constitution, Act, rule or regulation under which the paper has been laid;
  • Committees relating to the day-to-day business of the House:
(a) Business Advisory Committee recommends allocation of time for items of Government and other business to be brought before the Houses;
(b) Committee on Private Members Bills and Resolutions of the Lok Sabha.
(c) Rules Committee considers matters of procedure and conduct of business in the House and recommends amendments or additions to the Rules;
(d) Committee on Absence of Members
  • Committee on the Welfare of Scheduled Castes and Scheduled Tribes
  • Committees concerned with the provision facilities to members
(a) General Purposes Committee considers and advises Speaker! Chairman on matters concerning affairs of the House, which do not appropriately fall within the purview of any other Parliamentary Committee.
(b) House Committee deals with residential accommodation and other amenities for members;
  • Joint Committee on Salaries and Allowances of Members of Parliament
  • Joint Committee on Offices of Profit
  • The Library Committee
  • Committee on Empowerment ofWomen with members from both the Houses was constituted with a view to securing, among other things, status, dignity and equality for women in all fields;
  • Ethics Committee: On 4 March 1997, the Ethics Committee-of the Rajya Sabha was constituted. The Ethic Committee of the Lok Sbha was constituted on 16 May 2000.

Leaders of Opposition in Parliament

  • In keeping with their important role, the Leaders of Opposition in the Rajya Sabha and the Lok Sbha are accorded statutory recognition. Salary and other suitable facilities are extended to themthrough a separate legislation brought into force on 1 November1977.

Youth Parliament Competition

  • In order to develop democratic ethos in the younger generation the Ministry of Parliamentary Affairs conducts youth parliament Competition in various categories of schools and colleges! universities
  • The youth par1iament Scheme was first introduced in the School in Delhi in 1966-67.
  • Kendriya Vidyalayas located in and around Delhi were incorporated into the ongoing Scheme for Delhi Schools in 1978.
  • Subsequently, as separate scheme of youth parliament for Kendriya Vidyalayas at the National Level was launched in 1988.

Matters Under Rule 377 And Special Mentions

  • The Ministry of Parliamentary Affairs takes follow-up action on matters raised under Rule 377 of the Rules of Procedure and-Conduct of Business in Lok-Sabha and by way of Special Mentions in Rajya Sabha.
  • Also after ‘Question Hour’ in both the Houses of Parliament,Members raise matters of urgent public importance.

Administrative Set—Up

  • The Government of India (Allocation of Business) Rules, 1961 are made by the President of India under Article 77 of the Constitution for the allocation of business of the Government of India.
  • The Ministries/Departments of the Government are created by the President on the advice of the Prime Minister under these Rules.

Cabinet Secretariat

  • The Cabinet Secretariat is under the direct charge of the Prime Minister.
  • The administrative head of the Secretariat is the Cabinet Secretary who is also the ex-officio Chairman of the Civil Services Board.
  • In the Government of India (Allocation of Business Rules) 1961 ‘Cabinet Secretariat’ finds a place

National Authority, Chemical Weapons Convention

  • National Authority, Chemical Weapons Convention (CWC) was set u b a resolution of Cabinet Secretariat dated 5, May 1997 to fulfill the obligations enunciated in the Chemical Weapons Convention initially signed by 130 countries in a in a conference which concluded on 14 January 1993 for the purpose prohibiting of the development, production, execution, transfer, use and stockpiling of all chemical weapons by Member-States is a non-discriminatory process.

All India Services

  • Prior to Independence, the Indian Civil Service (UCS) was the senior most amongst the Services of the Crown in India. Besides the ICS, there was also the Indian Police Service.
  • A provision was made in Article 312 of the Constitution for creation of one ormore All India Services common to the Union and State.
  • After the promulgation of the Constitution, a new All India Service, namely, the Indian Forest Service, was created in 1966.
  • A common unique feature of the All India Services is that the members of these services are recruited by the Centre but their services are placed under various State cadres and they have the liability to serve both under the State and under the Centre.
  • Of the three All India Services, namely, the Indian Administrative Service (IAS), the Indian Police Service (IPS) and the Indian Forest Service (IFS), the Ministry of Personnel, Public Grievances and Pension is he cadre controlling authority for the IAS and Minister of authority for the IPS andMinistry of Environment & Forest for IFS.
  • The recruitment to all the three services is made by the UPSC. Administrative Reforms And Public Grievances
  • Department of Administrative Reforms and Public Grievances (DARPG) is the nodal agency of the Government for administrative reforms as well as redressal of public grievances relating to the States in general and grievances pertaining to Central Government agencies in particular.
  • The Department disseminates information on important activities of the Government relating to administrative reforms, best practices and public grievance redressal through publications and documentation. The Department also undertakes activities in the field of international exchange and cooperation to promote public service reforms.

Constitutional Provisions regarding the recruitment and service conditions of All India and Central Services.

  • The Constitution of India makes the following provisions with regards to recruitment and service conditions of All-India and Central Services:
  • Article 309 empowers the Parliament to regulate the recruitment and service conditions of persons appointed to Central Services and posts.
  • Article 310 provides that the members of All India Services and Central Services, and posts hold office during the pleasure of the President.
  • Article 311 provides that no member of-All India Services and Central Services and posts shall be dismissed or removed by an authority subordinate to that by which he was appointed. It further lays down that no such member shall be dismissed or removed or reduced to rank except after an inquiry in which he has been informed of the charges against him and given a reasonable opportunity of being heard in respect of those charges.
  • Article 312 authorises the Parliament to create new All-India Services (including an All-India Judicial Service) and regulate the recruitment and service conditions of persons appointed to these services. However they can be created by the Parliament only after the Rajya Sabha passes a resolution supported by two-thirds majority saying that interest to do so.

ARTICLES RELATED TO UPSC

Article No. Subject-Matter
Article 315 Public Service Commissions for the Union and for the States.
Article 346 Appointment and termof office of members.
Article 317 Removal and suspension of a member of a Public Service Commission
Article 318 Power to make regulations as to conditions of service of members and staff of the Commission.
Article 319 Prohibition as to the holding of office by members of commission on ceasing to be such members.
Article 320 Functions of Public Service Commission
Article 321 Power to extend functions of Public Service Commissions
Article 322 Expenses of Public Service Commissions
Article 323 Reports of Public Service Commissions

Administrative Reforms

(i) Annual Conference of the Chief Secretaries of State governments / UTs - To institutionalize the process of interaction with State governments /UT administrations on issues that relate to individual States / UTs as well as issues of common concern, the first conference was organized on February 1- 2, 2010. In the conference issues of science and technology and global issues impacting the government were also discusses.
(ii) The Civil Services Day - To rededicate and recommit ourselves to the cause of the people, has been declared as Civil Services Day and is being celebrated every year since 2006. The Fifth Civil Services Day was organized by the Department on April 21, 2010 at Vigyan Bhavan.
(iii)Conference of Secretaries of Administrative Reforms in State/UTs is held annually to share the experiences in the area of government reforms.
(iv) Regional Conferences on Administrative Reforms are held every year to disseminate and share best practices.
(v) Administrative Reforms Commission

Central Secretariat Services

  • The Central Secretariat has three services, namely, Central Secretariat Service (CSS), Central Secretariat Stenographers’ Service (CSSS) and The Central Secretariat Clerical Service (CSCS).

Union Public Ser Vice Commission

  • The Constitution provides for an independent body known as Union Public Service Commission (UPSC) for recruitment to Group ‘A’ and Group ‘B’ Gazetted posts under Central Government and for advice in various service matters.
  • The Chairman and members of the Commission are appointed by the President for a tenure of six years or till they attain the age of 65 years, whichever is earlier.
  • To ensure independence, members who were at the service of Government at the time of appointment are deemed to have retired from Government service on their appointment in the Commission.
  • The Chairman and members are also not eligible for further employment under the Government.
    They cannot be removed except for the reasons and in the manner provided for in the Constitution.

Staff Selection Commission

  • Staff Selection Commission SSC) initially known as Subordinate Service Commission was set upon I July 1978.
  • It has been entrusted with the work of making recruitment to
(i) all non-gezetted Group 'B' posts in the various Ministries/Departments of the Government and their Attached and Subordinate Offices which are in the pay scales of Rs 6,500- 10,500 and
(ii) all non- echnical Group ‘C’ posts in the various Ministries/ Departments of the Government and-their attached and subordinate-offices, exempted excepts those which are specifically exempted from the purview of the Staff Selection Commission.
  • The Commission is an attached office of the Department of Personnel and Training and comprises of a Chairman, two Members and Secretary-cum-Controller of Examinations.
  • The tenure of Chairman/Members is for five years or till they attain the age of 62 years, whichever is earlier.

Official Language

  • Article 343 (ii of the Constitution provides that Hindi in Devanagari script shall be the Official Language of the Union.
  • Article 343(2) also provided for continuing the use of English in official work of the Union for a period of 15 years (i.e. up to 25 January 1965) from the date of commencement of the Constitution.
  • Article 343(3) Empowered the Parliament to provide by law for continued use of English for official purpose even after 25 January 1965.
  • Accordingly, section 3(2) of the Official 1963 (amended in 1967) provides for continuing the use of English in official work even after 25 January 1965.
  • The Act also lays down that both Hindi and English shall compulsorily be used for certain specified purposes such as Resolutions, General Orders, Rules, Notifications, Administrative and other Reports, Press Communiques; Administrative and other Reports and Official Papers to be laid before a House or the Houses of Parliament; Contracts, Agreements, Licences, Permits, Tender Notices and Forms of Tender, etc.
  • In 1976 Official Language Rules were framed under the provisions of section 3(4) of the Official languages Act, 1963.

Committees/Samitis

  • A Committee of Parliament on Official Language was constituted in 1976 under section 4 of the Official Languages Act, 1963 to periodically review the progress in the use of Hindi as the Official Language of the Union and to submit a report to the President.
  • The Kendriya Hindi Samiti was constituted in the year 1967. It is chaired by the Prime Minister
  • It is the apex policy making body which lays down the guidelines for the propagation and progressive use of Hindi as Official Language of the Union.

Training

  • The Kendriya Hindi Prashikshan Sansthan was established on 31 August 1985.
  • The Central Translation Bureau was set up in March 1971 for translation having Headquarters in New Delhi.

Policy

  • In compliance with the Official Language Resolution, 1968, an Annual Programme is prepared by the Department of Official Language in which targets are set for the offices of the Central Government with regard to originating correspondence, telegrams, telex, etc., in Hindi.
  • A Quarterly Progress Report is called for from the offices of the Central Government regarding achievements vis-a-vis the said targets.
  • An Annual Assessment Report is prepared on the basis of the Quarterly Progress Reports, which is laid on the Tables of both Houses of the Parliament and copies endorsed to State Government and the Ministries/Departments of the Central Government.

Comptroller and Auditor-General

  • Comptroller and Auditor-General is appointed by the President.
  • Procedure and grounds for his removal from office are the same as for a Supreme Court Judge.
  • He is not eligible for further office under the Union or a State Government after he ceases to hold his office.
  • The President on advice of Comptroller and Auditor-General prescribes the form in which accounts of the Union and states are to be kept.
  • His reports on accounts of the Union and states are submitted to the President and respective governors which are placed before Parliament and state legislatures.

Administrative Reforms And Public Grievances

  • Ministry of Personnel, Public Grievances, and Pensions is the nodal personnel and administrative reforms agency in the country. It is headed, politically by the Prime Minister and administratively, by the Personnel Secretary.
  • In 1985,, a new full-fledged Ministry of Personnel, Public Grievances and Pensionswas created.With three separate departments i.e (a) Department of Personnel and Training; (b) Department of Administrative Reforms and Public Grievances; (c) Department of Pensions and Pensioner's Welfare.
  • The Various agencies which work under the administrative control of the Ministry are:
  • Union Public Service Commission (UPSC)
  • Staff Selection Commission (SSC)
  • Public Enterprises Selection Board (PESB)
  • Central Vigilance Commission (CV-C)
  • Central Bureau of-Investigation (CBI)
  • Central Administrative Tribunal (CAT)
  • National Academy of Administration (NAA)
  • Indian Institute of Public Administration (IIPA)
  • A Commission on Review of Administrative Laws was set up by the Department of Administrative Reforms and Public Grievances on 8 May 1998 with a view to identify proposals for amendment of the existing laws, regulations and procedures having inter-sectoral impact and also for repeal of all dysfunctional laws.
  • The Department set up a Second Administrative Reforms Commission AR in 2005 under the Chairmanship of Shri Veerappa Moily to prepare a detailed blueprint for revamping the pubic administrative system.

Reports of Second Administrative ReformCommission

  • Civil Service Reforms Second ARC in its 10th Report in Nov. 2008 on the subject “Refurbishing of Personal Administration has given various recommendation on Civil Services Reforms The recommendations include: (1) State of entry into Civil Services (2) Age of entry and number of attempts (3) Structure of Civil Services Examination (4) Othermodes of induction into Civil Services (5) Allotmen of cadres to All India Services.
  • First Report of ARC - ‘Right to Information: Master key to Good Governance
  • 2nd report of ARC: - Unlocking Human Capital: Entitlements and Governance - a case study
  • 3rd report ARC-crisis management
  • 4th report ARC- Ethics in Governance
  • 5th report of ARC-Public Order
  • 6th report of ARC-Local Governance
  • 7th Report - ‘Capacity Building for Conflict Resolution
  • 8th Report - Combating Terrorism
  • 9th Report - Social Capital-A shared Destiny
  • 10th Report - Scaling New Heights
  • 11th Report - Promoting e-Governance: The SMART Way Forward
  • 12th Report - Citizen Centric Administration
  • 13th Report- Organisational structure of Gout of India
  • 14th Report - Strengthening financial management systems
  • 15 Report - State arid district administration

Administrative Tribunals

  • The enactment of Administrative Tribunal Act in 1985 opened a new chapter in the sphere of administering justice to the aggrieved government servants. 
  • Administrative Tribunals Act owes its origin to Article 323-A of the Constitution which empowers Central Government to set-up by an Act of Parliament, Administrative Tribunals for adjudication of disputes and complaints with respect to recruitment and conditions of service of persons appointed to the public service and posts in connection with the affairs of the Union and the States.
  • As a result of the judgement dated 18 March1997 of the Supreme Court, the appeals against orders of an Administrative Tribunal shall lie before the Division Bench of the concerned High Court.
  • The Administrative Tribunals exercise jurisdiction only in relation to the service matters of the litigants covered by the Act.
  • The Act provides for establishment of Central Administrative Tribunal CAT) and the State Administrative Tribunals.
  • The CAT was set-upon I November 1985.
  • Today, it has 17-regular benches, 15 of-which operate at the principal seats of High Courts and the remaining two at Jaipur and Lucknow.

Inter-State Council

  • In a federal polity, in view of large areas of common interest and Shared action between the constituent units, coordination of policies and their implementation is extremely important Article 263 of the Constitution envisages establishment of an institutional mechanism to facilitate coordination of policies and their implementation between the Union and the State Governments.
  • In made by the Sarkaria Commission on Centre-State 4elations, the Inter-State. dated 28 May 1990.
  • Inter-State Council is a recommendatory body and has been assigned the duties of investigating and discussing such subjects, in which some or all of the States or the Union and one or more of the States have a common interest, for better coordination of policy and action with respect to that subject. It also deliberates upon such other matters of general interests to the States as may be referred by the Chairman to the Council.
  • Prime Minister is Chairman of the Council. Chief Ministers of all the States and Union Territories having Legislative Assemblies, Administrators of Union Territories not having Legislative Assemblies, Governors of States under President’s rule and six Ministers of Cabinet rank in the Union Council of Ministers, nominated by the Chairman of the Council, are members of the Council, Five Ministers of Cabinet rank nominated by the Chairman of the Council are permanent invites to the Council. The Inter-State Council has recently been reconstituted.

Governor

  • State executive consists of Governor and Council of Ministers with Chief Minister as its head.
  • The Governor of a State is appointed by the President for a term of five years and holds office during his pleasure.
  • Only ‘Indian citizens above 35 years of age are eligible for appointment to this office. Executive power of the State is vested in Governor.
  • Council of Ministers with Chief Minister as head, aids and advises Governor in exercise of his functions except in so far as he is by or under the Constitution required to exercise his functions or any of them in his discretion.
  • In respect of Nagaland, Governor has special responsibility under Article 37TA of the Constitution with respect to law and order.
  • Similarly, in respect of Arunachal Pradesh, Governor has special responsibility under Article 371 H of the Constitution with respect to law and order and in discharge of his functions in relation thereto.
  • Likewise, in the Sixth Schedule which applies to tribal areas of Assam, Meghalaya. Tripura and ““- Mizoram as specified in para 20 of that Schedule, discretionary powers are given to Governor in matters relating to sharing of royalties between district council and stage government.
  • Sixth Schedule vests additional discretionary powers in Governors of Mizoram Tripura in almost all Their functions (except approving regulations for levy of taxes and money lending by non-tribals by district councils) since December 1998.
  • In Sikkim, Governor has been given special responsibility for peace and social and economic advancement of different sections of population.

Council Of Ministers

  • The Chief Minister is appointed by the Governor who also appoints other ministers on the— advice of the Chief Minister.
  • The Council of Ministers is collectively responsible to legislative assembly of the State.

Legislature

  • For every state, there is a legislature which consists of Governor and one House or, two Houses as the case may be.
  • In Bihar, Jammu and Kashmir, Karnataka, Maharashtra, Andhra Pradesh and Uttar Pradesh, there are two Houses known as Legislative council and legislative assembly.
  • In the remaining states, there is only one House known as legislative assembly.
  • Parliament may, by law, provide for abolition of an existing legislative council or for creation of one where it does not exist, if proposal is supported by a resolution of the legislative assembly concerned.

Legislative Council

  • Legislative Council (Vidhan Parishad) of a state comprises not more than one-9f total number of members in legislative assembly of the state arid in no case less than 40 members (Legislative Council of Jammu and Kashmir has 36 members vide Section 50 of the Constitution of Jammu and Kashmir).

Manner of Election

  1. 1/3 are elected by the members of local bodies in the state like municipalities, district boards, etc.
  2. 1/12 are e1cted by graduates of three years standing and residing within the state.
  3. 1/12 are elected by teachers of three years standing in the state, not lower in standard than secondary school.
  4. 1/3 are elected by the members of the legislative assembly of the state from.

ARTICLES RELATED TO GOVERNOR

Article No. Subject-matter
Article 153 Governors of States
Article 154 Executive Power of State
Article 155 Appointment of Governor
Article 156 Term of Office of Governor
Article 157 Qualifications for appointment as Governor
Article 158 Conditions of Governors Office
Article 159 Oath or Affirmation by the Governor
Article 160 Discharge of the functions of the Governor in certain contingencies
Article 161 Power of the Governor to grant pardons and others.
Article 162 Extent of executive power of State
Article 163 Council of ministers to aid and advise the Governor
Article 164 Other provisions as to ministers like appointments, term, salaries, and others.
Article 165 Advocate-General for the State.
Article 166 Conduct of business of the Government of a state
Article 167 Duties of the Chief Minister regarding furnishing of information to the Governor, and so on
Article 174 Session of the state legislature, prorogation and dissolution.
Article 175 Right of the Governor to address and send messages to the house or houses of the state legislature.
Article 176 Special Address by the Governor.
Article 200 Assent to bills (i.e. assent of the Governor to the bills passed by the state legislature).
Article 201 Bills reserved by the Governor for consideration of the President.
Article 213 Power of Governor to promulgate ordinances.
Article 217 Governor being consulted by the President in the matter of the appointments of the judges of the High Courts.
Article 233 Appointment of district judges by the Governor.
Article 234 Appointments of persons (other than district judges) to the judicial service of the state by the Governor. amongst persons who are not members of the assembly, and
5. The remainder are nominated by the governor from amongst persons who have a special knowledge or practical experience of literature, science, art, cooperative movement and social service.

Legislative Assembly

  • Legislative Assembly (Vidhan Sabha) of a state consists of not more than 500 and not less than 60 members (Legislative Assembly of Sikkim has 32 members vide Article 371F of the Constitution) chosen by direct election from territorial constituencies in the state.

Reservation of Bills

  • The Governor of a state may reserve any Bill for the consideration of the President.
  • Bills relating to subjects -like compulsory acquisition of property, measures affecting powers and position of High Courts and imposition of taxes on storage, distribution and sale ofwater or electricity in inter- tate river or river valley development projects should necessarily be so reserved.
  • No Bills seeking to impose restrictions on interstate trade can be introduced in a state legislature without previous sanction of the President.

Union Territories

  • Union Territories are administrated by the President acting to such extent, as he thinks fit, through an Administrator appointed by him.

Administrative-Systemof Union Territories

Union Territories Executive Legislature Judiciary
1 Andaman and Nicobar Island Lt. Governor -- Under Calcutta High Court
2. Chandigarh Chief Commissioner -- Under Punjab and Haryana High Court
3. Dadra and Nagar Haveli Administrator -- Under Bombay High Court
4. Daman and Diu Administrator -- Under Bombay High Court
5. Delhi (a) Lt. Governor
(b) Chief Minister
(c) (Council of Ministers)
Legislative Assembly Separate High Court Court
6. Lakshadweep Administrator -- Under Kerala High
7. Pondicherry (a) Lt. Governor
(b) Chief Minister
(c) Council of Ministers
Legislative Assembly Under Madras High Court

Municipalities

  • The first such Municipal Corporation was set-up in the former Presidency Town of Madras in 1688; and was followed by similar corporations in the then Bombay and Calcutta in 1726.
  • While the Directive Principles of State Policy refer to village Panchayats, there is no specific reference to Municipalities except the implicitly in Entry 5 of the State List, which places the subject of local self-governments as a responsibility of the states.
  • In order to provide for a common framework for urban local bodies and help to strengthen the functioning of the bodies as effective democratic units of self-government, Parliament enacted the Constitution (74th Amendment) Act, 1992 (known-as Nagarpalika Act) relating to municipalities in 1992.
  • The Act received the assent of the President on 20 April 1993.
  • The Government of India notified 1 June 1993 as the date from which the said Act came into force.
  • A new part IX-A relating to the Municipalities has been incorporated in the Constitution to provide for among other things, constitution of three types of Municipalities, i.e., Nagar Panchayats for areas in transition from a rural area to urban area, Municipal Councils for smaller urban areas and Municipal Corporation for large urban areas.
  • In twelfth schedule 18 items are places within the purview of the municipalities. Some of important items are as follows:
  • Urban Planning including town planning.
  • Planning for economic and social development .
  • Roads and bridges.
  • Water supply for domestic, industrial and commercial purposes.
  • Public health, sanitation, conservancy and solid waste management.
  • Urban forestry, protection of the environment and promotion of ecological aspects.
  • Urban—poverty alleviation.
  • Vital statistics including registration of births and deaths.

Articles Related To Municipalities

Article No. Subject-matter
Article 243P Definitions
Article 243Q Constitution of municipalities
Article 243R Composition of municipalities
Article 243S Constitution and composition ofwards
committees, and so on
Article 243T Reservation of seats
Article 243U Duration ofmunicipalities, and so on
Article 243V Disqualifications for membership
Article 243W Powers, authority and responsibilities
of municipalities, and so on
Article 243X Powers to impose taxes by, and funds
of, the municipalities
Article 243Y Finance Commission
Article 243Z Audit of accounts of municipalities
Article 243ZAArticle 243ZB Election to the municipalitiesApplication to union territories
Article 243ZC Part not to apply to certain areas
Article 243Z-D Committee for district planning
Article 243ZE Committee formetropolitan planning
Article 243ZF Continuance of exiting laws and municipalities
Article 243ZG Bar to inference by—courts in electoral matters

Panchayats

  • Article 40 of the Constitution which enshrines one of the Directive Principles of State Policy lays down that the State shall take steps to organise village panchayats and endow them with such powers and, authority as may be necessary to- enable them to function as units of self government.
  • In the light of the above a new Part IX relating to the Panchayats has been inserted in the Constitution to provifor among other things:
  • direct elections to all seats in Panchayats at the village and intermediate level, if any, and to the offices of Chairpersons of Panchayats at such levels;
  • reservation of seats for the Scheduled Castes and Scheduled Tribes in proportion to their population for membership of Panchayats and office of Chairpersons in Panchayats at each level;
  • reservation of not less than one-third of the seats for women; and
  • fixing tenure of five years for Panchayats and holding elections within a period of six months in the event of supersession of any Panchayat.
  • In eleventh schedule 29 items are places within the purview of the panchayats. Some of important items are as follows:
  • Agriculture, including agricultural extension
  • Minor irrigation, water management and watershed development.
  • Social forestry and farm forestry.
  • Small-scale industries, including food processing, industries.
  • Drinking Water.
  • Rural electrification, including distribution of electricity.
  • Non-conventional energy sources.
  • Poverty alleviation programme.
  • Education, including primary and secondary schools.
  • Technical training and vocational education.
  • Health and sanitation, including hospitals, primary health centres and dispensaries.
  • Public distribution system.

Election Commission

  • Since its inception in 1954) and till October 1989, the Commission functioned as a single member body consisting of the Chief Election Commissioner.
  • On 16 October 1989 the President appointed two more Election Commissioners on the eve of the General Election to the House of the People held in November-December 1989.
  • However, the said two Commissioners ceased to hold office on 1 January 1990 when those two posts of Election Commissioners were abolished. Again on 1 October 1993, the President appointed two more Election Commissioners.
  • Simultaneously, the Chief Election Commissioner and other Election Commissioners (Conditions of Service) Act, 1991 was amended to provide that the Chief Election Commissioner and other Election Commissioners will enjoy equal powers and will receive equal salary, allowances—and other perquisites as payable to a judge of the Supreme Court of India.

Articles Related To Panchayat

Article No. Subject-matter
Article 243 Definitions
Article 243A Gram Sabha
Article 243B Constitution of Panchayats
Article 243C Composition of Panchayats
Article 243D Reservation of Seats
Article 243E Duration of Panchayats, and so on
Article 243F Disqualifications for membership
Article 243G Powers, authority and responsibilities of Panchayats
Article 243H Powers to impose taxes by, and funds of, the panchayats
Article 2431 Constitution of finance commission to review financial position
Article 243J Audit of accounts of panchayats
Article 243K Elections to the panchayats
Article 243L Application to union territories
Article 243M Part not to apply to certain areas
Article 243N Continuance of existing laws and panchayats
Article 243O Bar to interfere by courts in electoral mateers

RIGHT TO INFORMATION ACTS IN STATES

State Year of Enactment
1. Tamil Nadu 1997
2. Goa 1997
3. Rajasthan 2000
4. Kamataka 2000
5. Delhi 2001
6. Maharashtra* 2002
7. Assam 2002
8. Madhya Pradesh 2003
9. Jammu & Kashmir 2004
*Maharashtra repealed its earlier Right to Information Act of 2000 to bring out an improved one in 2002
  • The Act further provided that in case of difference of opinion amongst the Chief Election Commissioner and/or two other Election Commissioners, the matterwill be decided by the Commission by majority.
  • Independence of the Election Commission and its insulation fromexecutive interference is ensured by a specific provision under Article 324(5) of the Constitution that the Chief Election Commissioner shall not be removed from his office except in like manner and on like grounds as a Judge of the Supreme Court and conditions of his service shall not be varied to his disadvantage after his appointment.
  • The other Election Commissioners cannot be removed from office except on recommendation of the Chief Election Commissioner.
  • The term of office of the Chief Election Commissioner and other Election Commissioners is six years fromthe date he/she assumes office or till the day he/she attains the age of 65 years, whichever is earlier.

Amendments

  • The Parliament-on-22-March 2003 enacted the Election Laws (Amendment) Act, 2003 and Conduct of Elections (Amendment) Rules, 2003 which came into force with effect from 22 September 2003.
  • This Act also amended Section 77(1) regarding maintenance of election expenses by candidates whereby expenditure incurred by specified number of ‘leaders’ of a political party on account of travel by air or by any othermeans of transport for propagating programme of the political party alone shall be exempted from being included in the account of election expenses incurred by the candidate in connection with the election.
  • The Parliament on 1 January 2004 enacted the Delimitation (Amendment) Act, 2003 whereby Section 4 of the Principal Act was amended provide that the Delimitation will be held on the basis of the 2001 Census figures.
  • Parliament on 28 August 2003 enacted the Representation of the People (Amendment) Act, 2003 whereby open ballot system was introduced at elections to the Council of States.
  • In this system an elector who belongs to a political party is required to show the ballot paper after marking his vote to an authorised agent of that political party.
  • The requirement that a candidate contesting an election to the Council of States from a particular State should be an elector in that particular State was also dispensed with.

Electoral Reforms

  • In C.W.P. No. 4912 of 1998 (Kushra Bharat Vs. Union of India and Others), the Delhi High Court directed that information relating to government dues owed by candidates to the departments dealing with Government accommodation, electricity, water, telephone and transport (including aircrafts and helicopters) and any other dues should be furnished by the candidates and this information should be published by the election authorities under the Commission in at least two newspapers having local circulation, for information of electors.

Right To Information

  • In 2005, the Parliament has enacted a new legislation - the Right to Information Act (2005). This new Act replaces the old Freedom of Information Act, 2002, which was unnotified and hence, not operational.
  • The newlegislation confers on all citizens the right of access to the information and, correspondingly, makes the dissemination of such information an obligation on all public authorities. It aims at promoting transparency and accountability in the working of every public authority It has the widest possible reach covering Central Government, state governments, panchayati raj institutions, local bodies and recipients of government grants. Some important provisions are mentioned below:
  • It fixes a 30-day deadline for providing information; deadline is 48 hours if information concerns life or liberty of a person.
  • Information will be free for people below poverty line. For others, fee will be reasonable.
  • It provides for the establishment of a Central Information Commission and State Information Commissions to implement the provisions of the Act. They will be independent high-level bodies to act as appellate authorities and vested with the powers of a civil court.
  • The President will appoint a Chief Information Commissioner and governors of state will appoint state information commissioners. Their term will be five years.
  • The Act overrides the Official Secrets Act, 1923. The information commissions can allow access to the information if public interest outweighs harm to protected persons.
  • Its purview does not extend to intelligence and security organizations like Intelligence Bureau RAW- BSF,-CISF, NSG and so-on. However, information pertaining to allegations of corruption or violation of human rights by these organisations will not be excluded.